You are on page 1of 214
= med \ / ~USMLE Step1 Lecture Notes Behavioral Science Author and Executive Editor Steven R, Daugherty, Ph.D. Director of Education and Testing Kaplan Medial Rash Medical College ‘Chicago, IL Contributor, Alina Gonzalez-Mayo, PD. ‘Psychiatrist Executive Director of Curriculum Richard Friedland, M.D. Director of Publishing and Media Michelle Covello| Managing Editor Kathi MeGreeny Production Editor william Ng Production Artist Michael Wolf Cover Design Joanna Mylo Cover Art Rich LaRocco Chapter 2: Statistics (Chapter 3: Social Life in the United States... Chapter 4: Substance Abuse Chapter 5: Human Sexuality Chapter ‘Chapter 7: Defense Mechanisms Chapter 8: Psychologic Heath and Testing... ‘Chapter 9: Child Development ‘Chapter 10: Sleep and Sleep Disorders (Chapter 11: Physcan-Patient Relationships Chapter 12: Diagnostic and Statistical Manual IV (OSM IM) ‘Chapter 13: Organic Disorders Chapter 14: Pychopharmacology Chapter 15: Ethical and Lega ssues Learning and Behavior Modification 2+... 6 ° 238 97 10s na 1 wi 169 197 imédical Epidemiology Epidemiogy isthe study of the distribution and determinants of health-related sates within 2 population, + Epidemiology sees dscas as distributed within a group, not as a property ofan indi- vidal +The tools of epidemiology ae numbers Numbers in epidemiology ae ratios convert- cd into rates +The denominators key: who is "at rik’ for a particular event or disease sate Rates are generally, but not always, pet 100,000 by the Centers fr Disease Control and Prevention (CDC), but canbe per any mpl. Compare the numberof actual cases wit the number of potential eases to determine therate, ‘Actual cases __ Numerator Fotential cases ~ Denominator RATE A Incidence and Prevalence 1. Incidence rate the rate at which new events occur ina population. The numerator is the number of NEW events that occur in a defined period: the denominator isthe population at risk of experiencing this new event during the same period. [Number of new events in a specified period “Hamer af pesos “exposed to Hl” during thi psd Incidence rate Attack rate: # type of incidence rate in which the denominator is farther redced for some known exposure be, Focus on acute conditions 2. Prevalence rate ll persons who experience an event ina population. The numerator js ALL individuals who have an atribate or disease ata particular poiat in time (or during a particular perio of time) the denominator isthe population at risk of hav- ing the atibute or diseare a this point in time or midway through the period ____Alleases ofa disease ata given poiniperiod Prevalence re = <2T population a ak ate given pot or period“ 4, Point prevalence: prevalence at a specified point in time », Petiod prevalence; prevalence during a specified period or span of time The USMLE Requires You to Know. + The deitions ad use of res “idence and prevalence + Sandaried aes + Use and computations for screning es + How tidy bis in research * Common sudy desis Pts 1 USMLE Step 1: Behavioral Sciences 4. Prevalence can be changed by simply moving the cuto that defines what constitutes a case of disease. 20 uot % Provence 2sa% 145% fot 15) ‘Screened 85% Population 40% a 3.0% 15% 5 ° 50 60 70"-80..90 100 110 120 190 Diastole Blood Prossure (mm He) Figu 1-1. Prevalence of Hypertension 3. Understanding the relationship berween incidence and prevalence a. Prevalence = Incidence x Duration (P =1x) », “Prevalence pot” 4. Incident cass or new case are monitored ave time, fi Now cass joi pre-existing cases to make up total prevalence. fil Prevalent cases leave the prevalence pot in one of two ways: recovery cor death Incident Cases Mortality Figure 1-2, Prevalence Pot “able 1-1. Incidence and Prevalence [What happens to incidence and prevalence if Tneidence | Prevalence | Now effective rentent i initia? N t | New effcsve vaccine gains widesprend wet L v | Naber of pesons dying om the condition N v | increases? ‘Aditonl Federal research dollar a x N targeted oa speci condition? | Behavioral risk factors are reduced inthe v v | popslition at arge? T contacts between infeed pons and | noninfected persons ate reduced: | Rorairborne infectious disease? 4 L |___Foroninectious dest N N "Recovery rom the dieses more rapid han w v ita year ago Tonge survival tes forthe ease N 7 inreasing? Nivodange = darase? = ince 4, Morbidity rate rate of disease in 2 population at csk refers to both incident and. prevalent cases 5. Mortality rate ate of death in a population at risk; refers to incident cases only LUSMLE Step 1: Behavioral Sciences Disease course, # any, for 10 patients AIDS Cases 44-4 7 s-—4 *-—_} 441 10 svts00 swinoot Key: Onset Terminal Event Figure 1-3. Caloulating Indence and Preval 'B. Crude, Specific and Standardized Rates, 1, Cc sate actual meesured rate for whole population 2 Specific rate actual measured rate for subgroup of population, cg.,"age-speciic” or "Sevcapecfc™ rate, A crude rate can be expresiel asa weighted sum af age-speciic rates Each component ofthat sum has the following for (proportion ofthe population in the specified age group) (age-specific rate) 3 Standardize rat (or adjusted rat) adjusted to make groups equal on some factor ‘eg ages an “as if” statistic for comparing groups. The standardized rate adjusts or removes any difference between two populations based on the standardized variable This allows on “uncontaminated” or unconfounded comparison. Practice Question 1. Why does Population have a higher cade at of disease compared with Population C2 (ine Look at the age distribution.) ‘Table 1-2, Disease Rates Positively Correlated with Age r Population A Popalation 8 Population) I Ca Population | Ges Poplavon [Cases Population Downger [ana | 1,000 2 200 | 3.000 |_intermediate 211,000, 4 2,000 4 2,000 4 2,000 || Older 371,000 2 3,000 6 2,000 3 14,000 | “4 000 2 eo [10 000 cade Rates |rerno00 | 23 20 is “able 1-3. Types of Morelty Rates | Crd mori ate Deaths pr population " Cauespedic moray rte Death rom a specific cause pr i 1 population | Case-ftality rate ‘Deaths from a specific cause per umber of persons with the disease ‘roporttonate mori ate (PMR) | Deaths fiom spec cause per all deaths i | i | | | t Wutieal 5 Toole TRHEN TNH TP+TNGESEN “Tietoc pst Tico eps Fat poten Tce nto Understanding Sereening Tests 1. Sensitive the probability of correctly identifying a cas of disease. Sensitivity isthe ‘proportion of truly diseased persons inthe screened population who arc identified ‘a dieated by the screening test This is also known asthe “true positive rate” Sensitivity = TP/(TP + FN) = true postves/(trae positives + false negatives) 4. Measures only the distribution of persons with disease 'b, Uses data from the left column ofthe 2 x 2 table (Table 1-4) : T-senstvity = false negative rate 2 Specific: the probability of correctly identifying disease-free persons. Specificity is ‘the proportion of truly nondiseased persons wo are identified as nondiseased by ‘the Screening test. This is aso knovn asthe true negative rate” Specificity = TNITN+ FP) rue negatives/(true negatives + false positives) 1, Measures only the distribution of persons who ae disease-free i, Uses data from the right column of the 2 2 table specificity = fale positive rate 23, Bostve predictive valu the probability of disease ina person who rectves a postive test result. The probability that a person witha postive test isa true positive. (ie, has the disease is refered to asthe “predictive value ofa positive test.” Positive predictive value = TP/(TP + EP) rue positives(true positives + false positives) 4. Measures only the distribution of persons who receive a positive test result by. Uses data from the rop row of the 2% 2 table 4. Negative predictive value the probability of no disease in a person who receives ane ‘ative test result The probability that a person with a negative test isa true negative (Gexdoes not have the disease) is referred to as the‘predicive value ofa negative tes.” [Negative predictive value = TN/(TN + FN) = true negatives trae negatives + false negatives) ‘Measures only the distribution of persons who receive a negative test eslt ', Uses data from the bottom row of the 2X 2 table ‘5. Accuracy total percentage correctly selected the degree to which a measurement, oF fn estimate based on measurements, represents the true value ofthe atrbute thats being measured, Accuracy = (TP 4-TN)/(TP + TN + FP +N) rue poriives + true negatves)/total screened patients Practice Questions 1. What the ec of increased ineidene on sensi? On postive predictive vauet (None; screening does not ses incidence.) Wats the ec of increased prevalence on sensitivity On pose predictive valet (Sensitivity ays the sme, postive predictive value increases) Figure 1-4, Healthy and Diseased Populations Along a Screening Dimension 1, Which cutoff point provides optimal sensitivity? (B) Specificity? (D) Accuracy? (C) Positive predictive value? (D) 2, Note: point of optimum sensitivity = point of optimum negative predictive value ‘Point of optimum specificity = point of optimum postive predictive value D. Study Designs 1, Bias in research: devition from the truth of inferred results a Reliability ofa test to measure something consistently, either across testing situations (tet-etest ibility), within atest (spitchal? rebailty), or cross judges (inter-rater reliability). Think of the clustering of rifle shots at a target. Precision by, lit: dogroe to which a text messures that which was intended. Think of ‘marksman hitting the bulls-eye. Reliability is a necessary, but insufficient con- dition for validity. Accuracy 2, Types of bias ‘8. Selection bias (sampling bias: the sample selected is not representative of the population, Examples: 5. Predicting rates of heat disease by gathering subjects from a local health cab Epidemiology asa, 7 USMLE Step 1: imedical |i Using only hospital seeords 10 cstimate population prevalence (Berkson bis) li, People included ina study are different than thase who are not (nom respondent bss) ', Measurement bias: information is gathered ina manner that distorts the Information, Examples: “Measuring patients’ satisfaction with their respective physicians by using leading questions, eg, “You don ike your doctor, do you” fi, Subjects behaviors ltered because they are being studied (Hawthorne effet), Only a factor when there is no control group in a prospective study ¢ Experimenter expsctancy (Pygmalion effect): experimenters expectations inadvertently communicated to subjecs, who then produce the deste effet Can be aveided by double-blind design, where neither the subject nor the investigators who have contact with them know which group receives the inter vention under study and which group isthe control 4. Leatime bias gives a fale estimate of survival rates. Example: Patients seem role longer with the disease after itis uncovered by «sereening test. Actually, there is no increased survival, but because the disease is discovered sooner, pitlents who are diagnosed seem t ive longer ‘Recall bas subject fall to accurately recall events i the past. Example: “How ‘many times lst year did you kiss your mother” Likely problem in retrospective studies LatzJook bias individuals with severe disease are less Hikely tobe uncovered in ‘survey because they die first. Example:a recent survey found shat persons with AIDS reported only mild symptoms & Confounding bias: factor being examined is related to other factors of less interest, Unanticipated factors obscure relationship or make it som ike there isone when there is not. More than one explanation can be found for the pre- sented results, Example: comparing the relationship between exercise and heart disease in two populations when one population is younger and the other is older. Are differences in heart disease due to exercise oto age? “ble 1.5. Type of Bias in Research and Important Associations Typeofiins Definition ‘important Associations Bofations "Selection Sample not representative | Berksoas bias, noneespondent bias Random, independent sample Measurement Gathering the information | Hawtbore effect, Control groupiplacebo L disortsit group | fxperimenser expectancy) Researcher's bli affect, Pygmalion Double Blind design Tead-time Taily detection confused Benefis of screening ‘Measure"back-end” | vith increased survivat survival Recall Subjects cannot remember | Retrospective studies ‘Maple sources to accurately sonfirm information Tate look Severly diseased individvals | Early morality ‘Staily by severity sre not uncovered Confounding ‘Unantcipated actors Tidaden factors alfect results| Multiple stadies, obscure resus 4. Observational studies: nature allowed to tae its course, n0 intervention a. Case eport brief objective report of lineal characteristic or outcome fom, ‘single clinical subject or event,» = 1. Eg, 23-year-old man with teatment- resistant TH. No control group 'b. Cae series report: objective report of a clinical characteristic or outcome from ‘group of clinical subjects, 1... patient at local hospital with treatment- resistant TD, No control group . Cross-sectional study the presence or absence of disease and other variables ave determined in each member of the study population or in a representative “Sample ata particular time. The co-occurrence of variable and the disease can be examined i, Disease prevalence rather than incidence i recorded fi The temporal sequence of cause and effect cannot usually be deter mined ins eros-sectional study, who in the community now has teeatment-resistnt TB <4. Case-contol study: denies a group of people with the disease and compares ‘them with a suitable comparison group without the disease. Almost aways retrospective. Eg, comparing case of teatment-esstant TB with cases of non resistant TB. i. Cannot assess incidence or prevalence of disease 4, Cam help determine causal relationships ii. Very useful for studying conditions with very low incidence or prev «. Cohort sd: population group of those who have been exposed to rskfator i ‘entifed end flowed overtime and compared with a group not exposed tthe risk factor. Outcome is dscae incidence in each group, ¢g. following prison mate population and marking the development of treatment-esistant 7, good setarch design + Random eros unforurte but OK ard expected (a thea ta eb) + Systematic ero bad and bases resut (teat o vali) itedical 9 USMLE Step 1: Behavioral Sciences i. Prospective; subjects tracked foreard in time Ji, Can determine incidence and causal relationships i, Must fllow population long enough for incidence to appear cross- Sectional Case-Control cohort St __{ omen Figure 1-5. Diferentiating Study Types by Time 4, Analyzing observational studies + For cross-sectional studies: ue chi-square (22) + Forcohor studies: use relative tk andor attributable sk «2. Reatve risk (RR): comparative probability aking “How much more key?” |. Incidence rate of exposed group divided by the incidence rate of the ‘unexposed group How much greater chance does one group have of contracting the disease compared with the other group? ii. Bg. infant morality rate ia whites ie 8.9 per 1,000 lie births and 18.0 in blacks per 1,000 liv bits, then the relative risk of blacks versus whites is 1.0 divided by 89 = 2.02, Compared with whites black infants are twice as ey tod in the fis year oie Si, For statistical analysis, yields p-value be Autibusble risk (AR): comparative probability asking "How many more cases jn one group?” i. Incidence rate of exposed group minus the incidence rate of the unexposed group fi, Using the same example, atrbutable risks equal to 18.0 minus 8.9 = 9.1.0f evry 1,000 black infants, there were 9.1 more deaths than wete ‘observed in 1,000 white infints. In thie cae attributable risk gives the excess morality, iid Note that both relative risk and attributable isk tll s if there are dit ferences, but do nt tell us why those diflerences exis. For case-control studies: use odds ratio (OR) ‘. Qldds ratio: looks at the increased odds of getting 2 disease with exposure toa "bk factor versus nonexpostie to that factor i. Odes of exposure for eases divided by odds of exposure for controls ii The odds tata pecson with lung cancer was a smoker versus the ode that a person without lung cancer was a smoker “Table 1-6, Case-Control Study: Lung Cancer and Smoking Luang Cancer | No Lung Cancer Smokers 659 (A) 984 (B) Nonsmokers 3 348 (D) fi, Odds rat BD J. Use OR = AD/BCas working formula ‘or the above example 659 348 BC OR X IS vi The odd of having been a smoker are more than nine times greater fot someone with lang cancer compared with someone without lng cancer. Practice Question How would you analyze the data from this case-conteo study? “Table 1-7. Case-Control Study: Colorectal Cancer and Family History Practice — ELT ‘No Colorectal Colorectal Cancer Caner TOTALS Family History of TT Colonel Gancee uo @ _|w i | No Fama tistory of | | Colorectal Cancer | 200 20 220 rors a 0 | ay (eoniz0o) ANSWER: BC (120)(20) OR vi, ‘This means tha the odd of having a family history of colorectal can= ‘erate five times greater fr thove who have the disease than for those ‘who do not Epidemiology iiédical 11 LUSMLE Step 1: Behavioral Sciences ‘Tle 18, Differentiating Observational Studies ‘hmacrsie | Cr Seonal Suis [Gave ComolSidi | Gahort tudes Tae Onesie pin Tepe — Incidence NO NO ‘YES | rele 1 NO xd cus xo vs ~ | Tole of nue esr de veg wi done Tadwih dene | ‘ees ‘section fi ator” | ny vik for | Sg i nor acing and disease single disease ‘many diseases | Daan Giepucwanen | Clowes | Reiko oinae | 12 médical ‘5. Clinical trials (intervention studies: ressarch that involves the administration of a test regimen to evaluate ts safety an eficacy 4 Control group: subjecs who do nat receive the intervention understudy; used 252 source of comparison to be certain thatthe experiment group is being affected by the intervention and not by other factors. In clinical teas hiss most often a placebo group. Note that control group subjects must be a imi laras possible to intervention group subjects. b. For Food and Drug Administation (FDA) approval, three phases of clinical tei als must be passe. 1, Phase One: testing safety in healthy volunteers ii. Phase Tivo: esting protocol and dose levels ina small group of patient volunteers lik Phase Three testing effcaey and occurrence of side effects ina larger _roup of patient volunteers. Phase Il isconsiered the definitive test «¢ Randomized controled clinical trial (RCT) i. Subjects in the study are randomly allocated into “intervention” and control” groups to receive or not receive an experimental preventive ‘or therapeutic procedure or intervention. Hi. Generally regarded as the most scientifically rigorous studies avail- thle in epidemiology Ih, Double-blind RCT isthe type of study Feast subject tobias, but aso ‘the most expensive to conduct. Double-blind means that nether sab jects nor researchers who have contact with them know whether the subjects are in the treatment or comparison group. 4. Community tra: experiment in which the unt of allocation to recive a pre- ‘ventive or therapeutic regimen ie an entire community or political subdivi- son, Docs the treatment workin rel world circumstances? All subjects receive intervention, but at cifeent times: tical reasons, ne group involved can remain untreated, Eg. AZT trials. Assume double-blind design. Group A receives AZT for 3 ‘months, Group Bis control. For second 3 months, Group B receives AZT and Group A is control (Chapter Summary ‘pideiolgy the toy ofthe ate of dean a popation. Rate = atuapotentat Indéence=NEW cases Prevalence = ALL cases Freee incidence x Dutton Ssaeeing est caeulion re: “Tues ontop and die by everthing” “Sensi detection of disease = T/TP+PN Speci iden the heathy = TNTNSFP Posie predictive vale = of poses tha are ly diseased = Te/TPOFP ‘Negatvepredive value= of negates that are ry nondisesed = TNT. curacy =how onthe test sight overall = TP» THYTP + TN» FP «FN Important ype f bias ae sletian, measurement, expel, leadtime, real helook, ond confounding. Obserional ties: Case repo ‘Single patient caseseties Sever patents osssectona (ne point in tine Cesecono Backvard in time, odd ato ohert Forardin tine, late rk and atbatable isk ervetion Studies: Cina tis The vimate tes, Doulebind designs Nether subjects nar researches inow Communi als “esting eae? sso tials No uiveated paricpans ) | Statistics A. Key Probability Rules 1, Independence: across multiple events ‘a. Combine probabilities for independent events by mulhipliation i. vents ane independent ifthe occurrence of one tells you nothing about the occurrence of another. The issue heres the intersection of two set ii, Equi the chance ofhaving blond hai is 0.3 and the chance of having ‘cold is 02, the chance of mevtinga blond-baired person with a cold 0.3 x02 = 0.06 (or 6%) b evens ate nonindependent i. Multiply the probability of one event by the probability ofthe second, sssuming thatthe fst has occurred, fi, Egaifabox hae five white balls and ive Blackball the chance of pick- ing tra Black balls i (5/10) x (4/9) = 05 x 0.44 = 0.22 (or 2254) 2, Mutually exclusive: within a single event ‘4 Combine probebilies for mutually exclusive evens by addition i. Mutually exclusive means that the eceurrence of one event precludes the occurrence ofthe other, The issue here isthe union of tw sets ii, Eqs fa coin lands on herds, it cannot be tals the two are mutually esclusive. If coin ie Bipped, the chance that it wil be either heads or tails is: 05 + 05 = 10 (or 100%) b, Wwo events are not mutually exclusive i, The combination of probabilities is accomplished by adding the two together and subtracting out the multiplied probabilities. fi, Ege ifthe chance of having diabetes is 10% and the chance of being ‘bee is 30%, the chance of meeting someone who is obese or had di betes or both is 0.1 + 0.3 ~(0.1x03)= 037 (or 37%) Note ‘The USMLE Requires You to Kot: + fais sovesing joint probably ‘Types of cental tendency + Measutes of variation + Constant parcentages under the poral cone + Computing ad using canfidence inten, induding serdar rors and scores, + Types of sales The gio asic, including pates and Type and Type eros + Howto chose ameng very brs statistical tess. ou do rat need to kno al of Stasis, but jst enough to araner the presented uestions médical 15 USMLE Step 1: Behavioral Sciences OO Mutualy Exclusive Nonmutually Exclusive Figure 2-1. Venn Diagram Representations of Mutually Exclusive and Nonmutualy Exclusive Events ctice Questions 1, Ifthe prevalence of diabetes is 198, what isthe chance that three people selected at random from the population wil ll have diabetes? (0.1 X 0.1 0.1 = 0.001) (Chicago has a population of 10,00,000. 25% ofthe population is Latino, 3096 is African America, 586 is Arab American, and 40% are of European extraction, how many people in CCicago are casted as other than of European extraction? (25% + 3055 + 58% = 6006 60% X 10,000,000 = 6,00,000) 3. Atage 65, the probability of surviving forthe next 5 yeas i 0.8 fora white man and 0.9 for ‘white woman. For a married couple who are both white and age 6, the probability thatthe wife wil bea living widow 5 years ater (a) 9096 @) 72% (©) 1896 (©) 10% &) 9% Answer: C. This question asks for the joint probability of independent events; therefore, the probabilities are multiplied. Chance ofthe wife being alive: 99%, Chance oft husband being ead: 10086 — 80% = 20%, Therefore, 0.9 X 02 = 18%. 16 medical 4. Ifthe chance of surviving fo 1 year afer being diagnosed with prostate cancer is 80% and the chance of surviving for 2 yeas is 60%, what is the chance of surviving for 2 ears given thatthe patient i alive atthe end ofthe fst year? (a) 208 ©) 48% © ow © 5% © am Answer: D. The question tests knowledge of “contional probability” Out of 200 patients, $0 are ative a the end of I yearand 60 tthe end of year wo, The 60 patent alive ater 2 yearsaré a sub sctof those that make itt the frst yar. Therefore, 6/80 = 75%. 8B. Descriptive Statistics: Summarizing the Data 1. Distributions ‘Statistics dels with the world a distibutions. These disebutions ae summarized by @ ‘central tendency and variation around that center. The mos important distribution isthe normal or Gaussian curve. This "bel shaped” curve is symmetric, with one se ‘the mirror image ofthe other. Symmetic Ms x Figure 2:2, Measures of Central Tendency 4. Central tendency isa general term for several characteristics ofthe distribution of| ast oF values or measurement around a value ator nest the middle ofthe set '. Mean Qi}: synonym for average. The sum of the values of the observations ded bythe oumbers of observations . Median (Ma): the simplest division ofa set of measurements is into two parts — the upper anc lower halt The point onthe scale that divides the group inthis ‘way is the median. The measurement below which half the observations fll: the Bath percentile 4, Mode: she most frequently occurring value in set of observations Statistics aay " LUSMLE Step 1: Behavioral Sciences, Given the distribution of numbers 3, 67,79, 10,12, 15,16 "The mode is7, the median is 9, the mean is 94 Skeyed curves not all curves are normal. Sometimes the curve is sewed ther positively or negatively. A positive skew has the alto the right andthe mean ‘greater than the median, A negative skew has the talo the lft and the median sreater than the mean For skewed distributions, the median isa beter represen tation of central tendency than i the mean, Negatively skewed Poatively skewad Xa Max Figure 2-3, Skewod Dietrbution Curves 2, Measures of vari "The simplest measure of variability is the range, the difference between the highest and the lowest score. But the range is Unstable and changes easly. A more stable and more useful measure of dispersion isthe standard deviation 2, Tocaleulate the standard deviation, we fist subtract the mean from each sore to obtain deviations from the mean. This wil give us both postive and nega tive values. But squaring the deviations, the next step, makes them all positive. The squared deviations are added together and divided bythe numberof cases. ‘The square root is taken ofthis average anc the result isthe standard deviation Gors0), ‘The square of the standard deviation (6) equals the variance 18 inédical Figure 2-4, Comparison of Two Normal Curves with the Same Means, ut Diferent Standard Deviations Figure 2-5. Comparison of Three Normal Curves with the Same Standard Deviations, but Different Means You wil not be asked to calculate a standard deviation or variance onthe exam, bout you da need to know what they areand how they eat tothe normal curve In ANY normal curve, a constant proportion ofthe cases fll within one, two, and thre standard deviations ofthe mean, 4. Within one standard deviation: 68% fi, Within two standard deviations: 95.556 ii, Within three standaed deviations: 99.796 sor%, + 95.5% ‘ 0.15% 24% 135% 36 sie | asm | 2am 0.1586 ~a8 2 ats x ws as 436 Figure 2-6, Percentage of Cases Within One, Two, and Three Standard Deviations of the Mean in @ Normal Distribution Know the constant presented in Figure 2-6 and be able to combine the given constants to answer simple questions Practice Questions 1. Ina normal distbution curve, what percent ofthe cases are below 2s below the meant (5%) 2. Im a normal distribution curve, what percent of the cases are above 1s below the mean? (8896) 3. A student who scores a the 97.5 percentile fll where on the curve? (2s above the mean) 4. A student took two tests Som = Mean atd Deviation Test A 45 30 5 Test B o 0 10 (On which test di the student do beter? (On Test A, she scored 3s above the mean versus only 2s above the mean for Tet B.) Tet A C. Inforent Statistics: Generalizations From a Sample to the Population asa Whole “The purpose of inferential statistics et designate how likely iti that a given finding is simply the result of chance. inferential satsties would not be necessary if investigators studied all members of a population, However, because we can rarely observe and study entire populations, ety to sclect samples that are representative of the entire population s0 that we can general- {ae the results from the sample othe population. 1, Confidence intervals Confidence intervals are a way of admiting that any measurement from a sample is only an estimate of the population, Although the estimate gives fom the sarspeis ike Ty tobe close, the true wales for the papulstion may be above or below the sample val tus, A confidence interval speifies how far above or below a sample-based value the population valu ies witht a given range, from a possible high to a posible low. The true mean, therfore, i most likely tobe somewhere within the specified range Confidence interval ofthe mean ‘he confidence interval contains two parts: ‘An estimate ofthe quality of the sample forthe estimate, Known asthe stan- dard error ofthe meal bs, The degre of confidence provided by the interval specified, known a the stan dard ot Z-score, «, The confidence interval of the mean can be caleulated by I i Mean = appropriate Z-score x standard eror ofthe mea 4, Standand error of the mean isthe standard deviation divided by the square root of the sample size i. Ifthe standard deviation is larger, the chance of ezor inthe estimate ie gree. fi if the sample sz is anger the chance oferor inthe estimate isles, « Zescore or standard score is a score from a normal distribution with a mean of| (and a standard deviation of 1, Any distribution can be converted into a 2: score distribution using the formula: [z=] wlio «fae aes i nn Zescores are used in computing confidence intervals to set th level of conf dence, Recall that in a normal cistibution, 95.5% of the cases are within two standard deviations (2) of the mean. To get 95% confidence and 99% con dence, ll we need to know is what symmetric Z-sore to use to contain exactly 959% ancl 99% ofthe cases, 4, Bor 95H confidence = 1.95 for calculation purposes, use Z-score of 2.0 USMLE Step 1: Behavioral Sciences fi, For 99% confidence = 258 {or calculation purposes, wse Z-core of 25 fi, Note that 99% confidence interval wil be wider than a 95% interval Practice Questions 1. Fory-nine women, aged 15 to 25, wer selected ina random sample ofa local neih- borhood. Each woman was given a full physical exam, inluding measurement of ‘weight. The results show the group hasa mean weight of 1405, with standard devi- tion of 35, Specify the 9560 confidence interval fortis sample. Answer: 140 = 20 x (35/V49) = approximately 139 0 141. This tells us that we are ‘9599 sre thatthe te mean (if we measured the whole population) restserween 139 and ial 2. Assuming the graph (Figure 2-7 presents 9586 confidence intervals, which groups if any, are statistically diferent from eachother? High end I Prossure Low Drug A Ong 8 Drug ¢ Figure 27. Blood Pressures at End of Clinical Trial forThree Drugs Answer: When comparing two groups any overlap of confidence intervals means the groups ate not significantly different. Therefor, if the graph represents 95% conf dence intervals, Drugs B and Care no diferent in their effects; Drug B sno different ‘om Drug A; Drug A has a better effect than Drug C 3. Confidence intervals for relative risk and odds ratios, If the given confidence interval contains 0, then there is no statistically significant eflet of exporue Example: | Relative Risk Confidence Interval Interpretation 1s? (12-225) ‘Statistically significant (nereased sk) 165 (09-234) [NOT statistically significant (cskis the same) 076 (056-093) Statistically significant (decreased risk) . Nominal, Ordinal, Interval, and Ratio Scales ‘Table2-1. Types of Scales in Stat Te convert the world into numbers, we use four types of sales. ‘Type ofSeale | Key Words Nominal (Categoria) Diferent groupe “his or tar or at Gender, comparing ‘mong teatent ‘Oxsnal Groups in sequence Comparative ual, | — Olympic medals dass rank order rank in media school, Interval Trac differences among Quant meas and “Height, weight blood soups standard devation pressure drug dosage _| Ratio Totes + ee eo point Zero means 20 Temperature measured indegtes Kevin | 1, Nominal o categorical sale ‘A nominal scale pats people nto bones, without specifing the relationship berween thebons, Gender sa common expt ofa nominal sale wih two groups mae and female, Anytime you ca sy 1slther thio that you are dealing with nominal sl, Othe examples es drug versus control group 2. Ordinal cle Numbers can ao be wed to expres ordinal or rank order relations. Fr example we say Bens taler han Fred Now we know mote than just the category in which o pace Someone. Weknow something about the relationship bxtwexn the categories (qui). What we do ot knw is how fret the wo categories are (quanti), Class ank in soda schol and modal tthe Olympics ate examples of ordinal cals Note 3: Interval acle Theses ar hero ‘es ale graded in equal incremens In the sale of length, weknow that one inch ranged fm ast is equ to anyother inch. Interv saes allow us to ay not only that things are pfomaton poe diferent, but by how much, IF «measurement has a mean and a standard deviation, treat it asa interval sae, Is sometimes called a “numeric sale” 4 Ratio sale ‘The best measure isthe ratio sale. This scale orders things and contains equal intr ‘als lke the previous two sales, But it also has one additional cuality: a true zero point. In ratio sale, 270 is floor, you cat go any lower. Measuring temperature ‘sing the Kelvin sale yields ratio scale measurement. (pominal to es information provided to. scale an be degraded to a aver scale, eg inten dat can be tweed as ona For the USM, conertrete on enthingsoinal and interval scales. iédical 25 UUSMLE Step 1: Behavioral Sciences, . Understanding Statistical Inference "The goal of science isto define reality. Think about statistics asthe referee in the game of si ‘ence, We have al agreed to play the game according to the judgment cll of the referee, even though we know the zeforee can and willbe wrong sometimes. 1, Basie steps of staal inference Define the research question: what are you trying o show? ', Define the null hypothesis, generally the opposite of what you hope to show i, Nall hypothesis says thatthe findings are the result of chance or ran- dom factors. you want to show that a drug works, the nul pothesis will e tha the drug dace NOT work Ji, Alternative hypothesis says what isft after defining the ml hypoth- ‘ss Im tis example that the drug does actually work . Tho ypes of ml hypotheses |. Ono-taled, ie, directional or “one-sided,” such that one group is ther greater than, or less than, the other Eg Group A is not < than Gronp By ar Group A is not > Group B 4, ‘Two-tiled, i, nondirctional of “two-sided? such that two groups are not the sme. E., Group A= Group 8 2, Hypothesis esting ‘A his point data are collected and analysed by the appropriate statistical test. How to run these tests snot tested on USMILE, but you may need tobe able to interpret results ‘of statistical ests with which you are presente 1 p-Value: to interpret output from a statistical test focus on the p-value. The ferm pevalue refers to to things. In its fist sense, the p-value isa standard against which we compare our results. Inthe second sense, the poalue is a result of computation. {, ‘The computed p-value e compared with the p-value criterion to test i statistical significance. fhe computed value isles than the criterion, ‘we have achieved statistical significance. In genera, the smaller the p the better, 4 The p-value criterion is traditionally set at p < 0.05, (Assume that these are the criteria if no other vai i explicitly specified.) Using this standard: + If p:$0.05, reject dhe ml hypothesis (eached stats significance) + IE p> 0.05, do not rejet the null hypothesis (has not reached stats- tical significance) 24 medical Note i Possible (9 =0.13 (computed p valve) We ces ara core S ono a tea weeecrert falta eet it Synge DZ (MZ = 6036, DZ = 30%) |b, Marked ethnic-group difrences Asians, Jewish American and Italian Americans ‘much es ikely to develop alcoholism than Americans with northern European Capacity to tolerate alcohol isthe hey (enzyme induction, lack of tyrosine kinase) 4. biologic father was an alcoli, the incidence of sleohoism in males adopt «into nonaleaholic families is equal tothe incidence of alcoholism in sons ‘sed by biologic aleoholic fathers. «. Family history of alcoholism increases likehood of unipolar (major) depres- sion in offspring. 11, CAGE questions 1. Have you eve tried to Cut down on alcoho intake and not succeeded? 1, Have you ever been Annoyed about criticism concerning your drinking? «Have you ever felt Guilty about your drinking behavior dL Have you ever had to take a drink as an Eye-opener inthe morning to relieve the anxiety and shakinesst 12, Medical complications of alcohol abuse icthoss, alcoholic hepatitis, pancratis gastric or duodenal ulcer, esophageal varices, middle-age onset of diabetes, gastrointestinal cancer, hypertension, peripheral ‘neuropathies, mpopathies, cardiomyopathy, cerebral vascular accidents, erectile dys fiction, vitamin deficiencies, pernicious anemia, and brain disorders, including ‘Wernicke-Korsakof syndrome (mortality rate of untreated Wernicke is 50% treatment is with thiamine) 1}, Chronic alcohol use can change genes related to myelin in the fontal cortex. médical 51 LUSMLE Step 1: Behavioral Sciences “Table 41, Common Laboratory Value Changes for Alcoholics ‘Alanine aminotansievase (ALT) Teease “Aspartate aminotransferase (AST) increase Amylase increase ie acid ” increase Tilyeeride increas Cholesterol increase Calcium decrease decrease decrease Platelet count decrease Hematocrit decrease 14, Treatment issues 2, Most successful way to get person into treatment: referral by employer Alcoholes Anonymous largest source of aleohol treatment in US. ‘The original 12-sep program. Grassroots movement Believed to be mast suecesfl treatment Provides substitute dependency, social support, inpization and hope, and external reminders that drinking is aversive: Meetings and sponsors Least expensive treatment Al-Anon fr farily and fiends: deals with codependence and enabling behaviors 15, Pharmacologic treatments a, Dislfram (Antabuse) Decreases aleohol consumption Interaction with aleohol produces symptoms of nausea chest pain, bbypervenilation, tachyeardia, vomiting ii, Effective for short-term treatment only [ust be used with pychotherapy (or 12-sep progeam) Disutram inne Alcohol ——p= Acctaldchydo, p> Acetic acid (When accumttod, causes nausea and hypotension) ‘Acohel dehycrogenase Aldehyde dohydrogenase Figure 43. Disulfiam Troatment be. Acamprosate i. Helps prevent relapse AA Lowers the activity of the receptors for glutamate (chroaie alcohol sbuse incresss the number ofthese receptors) i, Effects persist ale treatment ended i Not widely used «& Benvodiavepines(e@, lorazepam) iL Helps prevent seizures li, Seizures can oceur during periods of heavy drinking or during detox ‘cation 4. Nalteesone i. Given to recovering scobolics i. Reduces craving, Drinks don't taste good. Helpe them stop after fst drink ii, On naltrexone elapse~50%6 if not, relapse % imédical 53 (ponusuey) ror oo odes poe aussi scjne preset ‘vayo‘uintdy Gong USMLE Step 1: Behavioral Sciences ‘sang soumrsgng Jo A2eurUNG 2-F 19H, ” Substance Abuse uo 20 ods Spensun iiiédical USMLE Step 1: Behavioral Sciences . Other Abused Substances estayy (MDMA) 4. Acts as hallucinogen combined with an amphetamine ’. Effects begin in 45 minutes and last 2 to 4 hours. «& Use spreading beyond “rae” partis 4 Dehydration may be a problem ‘Fatigue the day afer use Long-term use effects i. vidence of destruction of serotonin receptors (increased impulsiveness) li, Destruction of connections berween brain ells (memory gaps) 2. Anabolic steroids 4. Taken by male and female athletes to increase performance and physique be, With chronic use, can cause cardiomyopathy, bane mineral loss with ate osteo porosis hypertension, diabetes, mood lability, depression, atypical psychosis «. Presenting signs include skin atrophy, spontaneous bruising. acne low serum potassium levels i. For men: breast development, seroal pain, premature baldness ii. For women: disrupted menstrual cee, deepening of voice, excesive body hair 3. Summary points 4 Only nisotine does not have intoxication b, Hallucinogen, inhalants, cannabis and PCP do not have withdrawal reactions Schizopheniform delesonal disorder: amphetamines, cocaine, hallucinogens, «cannabis, and pheneylidine coorinelamphetamine intoxication cocancamphetamine withdrawal | Archytimias cocaine intoxication Violence Por (Vertcalayatagmus | PCP | Pinpoine papi plate overdose_(Weatment~ naloxone) | ‘opiate withdrawal fwcetment = clonidine) | ike | oblate withdrawal ucetment = clonidine) | ashbacks 1. | Seiares benaci withdrawal 4. Epidemiology 4 Physicians tend to underdiagnose substance abuse problems ofall ypes i. Detection eats: 25% for surgery and OB/GYN, 659 for psychiatry Ji. Women, high SES patients (inclding other physicians) likly to be uunderdiagnosed ', Most illicit drug users are employed full-time. «About 3396 of psychiatric disorders are substance abuse disorders i. Men outnumber women roughly 5 times. ii Prevalence of substance abus in newiy admitted psychiatric inpatients ‘or outpatients is roughly 50%, i, These “dual diagnosis” patients are very dificult to treat and tend to continue use when on inpatient wards 4. Substance abuse adds to the suicide risk of any underving pychiatric diagnosis «50% of emergency department visits are substance related D, Substance-Abusing Physicians 1. Pachiatrists and anesthesiologist ave highest rates 2. Physician impairment issues aze deat with y the State Licensing Boards. 3. Ifyou suspect that colleague has a substance abuse problem: 2 You must report ito hospital administration by. Get the colleague to suspend patient contact, «Telly, get the colleague into treatment. Chapter Summary The NAC dopa pathways the pinay adcton pataay in the rin ‘cao and Noho tough ad sesh mot coy th probl nteUS, oc se ass mare seats Aes isthe mes abs ug or alge + Tse US ots have dining rote. + al aotlsndome (4) the eng noun case of mera recto owed by Down sion + Theeisincesngeiderce ca sto gene cotratn to ahs ston by ‘once eee ad aopton des + Capac toler ach san mortar pedsping aortas + Mos fein wo tan aos foc veamereferlFom an enpler Mos sucess mode of resnent Abi Aronynas 29 pgm + Dg seo tet aan: dutian, samp, beatin nd netraane ter used Substances Rese able 42: Ha sstrces of abs. sis and rms iton nd wa _mades af action, and helpful associations. ‘Physicians tend to undesdiagnase substance abuse, particularly among women and high SES individuals. About tio he psi dds nda he Eis ru to ug bse, met cameo bye. Ales report substance abusing physcars | Human Sexuality A. Sexual Behavior in the United States 1, Adolescent sexual behavior ‘4 Nearly 70% ofall unmarried females are nonviegins by age 19 (80% of males). 4. 280 of females have had sex by age 15; almost all these encounters were forced or coerced, i. Average age offre sexual experience: 16 1, Adolescents in the aggregate sil drift into sexual activity rather than decide to have sex i, Most adolescent sexual activity still ake place in the cntext of one imary rationship. ii, Most adolescents are not promiscuous, bu “serially monogamous Recent survey: 57% of adolescent claim to ave used a condom the las time they had sex i, Other research suggests that they do not do as they say. fi, More than 50% of sexually active adolescents do not use birth contol regulary, 2, Teenage pregnancy ‘About | milion US. tensgers become pregnant each year, 1. 10% ofall teenage gis i, 50% ofall unwed mothers are teenagers, by, 5086 have the chil 4, 3396 have lective abortions. fi Theremaindor are spontaneously aborted, «About 33% of gts aged 15-19 have at least one unwanted pregnancy Single mothers account for 70% of births to girls aged 13-19. e, Consaquences of teenage pregnancy i. Formother: + Leading cause of school drop out + High isk for obstetric complications i. For chil: + Neonatal desths and prematurity are common. + Possible lower level of intellectual functioning + Problems of single-parent family (increased sk of delinquency sui ide) Note ‘The USMLE Requires You To Kou: + Seal behavior in the United States * Paphilis + Dierental agros among sexu dysfunctions + Fads abou sem paces LUSMLE Step 1: Behavioral Sciences “Table 5-1. Male Sexual Response Cycle 3. Sexual iy transmitted diseases (One in five teenagers wil havea sexualy transmitted disease: rates for gonor shea and chlamydia are higher for adolescents than for any older group, Highest incidence: most common sexually cransmited disease is uman papi Joma virus (HPV). Highest prevalence: one in five Americans has herpes simplex virus, type 2 (SV) |, Chlamydia isthe most commonly reported STD in women. fi, Gonorthea i the most commonly eported STD in men. Syphilis (primacy and seconds): 4, Cases have doubled since 1970 fi, Rate now more than 20/100,000 Gonorthes i, Number of cass as declined by half since 1975 ii, Since 1975 increase in resistant stains ‘Excitement Phase ‘Orgasm Phase | Resolution Phase 7 ‘Sexual flush Sisseconds —_| Disappears Penis | Vasocongestion, penile erection | Ejaculation Detumescence [[Serotum [Tightening and ling No change Decrease to baseline size [Reses | Elevation and increase in size | No change Decrease to baseline size descent [Breasts [Nipple erection No change I Dring the excitement and ongesm phase, ther ean increase in respiration, tachycardia up to 180 beats per minute ars in systolic blood pressure 20-100 mm Hg and diastolic blood pres sure of 10-50 mm Hg. lical Table 5-2, Female Sexual Response Cycle ‘Body Area | Excitement Phase ‘Orgasm Phase | Resolution Phase ] Skin Sexual ash 315 seconds [Disappears | Besse Nile wenn arate | May ese | Rear nara | Glitoris | Enlargement, shaft retracts ‘No change Detumescence, shaft returns (0 “jain Naloparoasceateandfaton | Nochange | Neloparoar nese to normal majors | Muktprous: congestion cee tnd elena Matipaous decree to normal Labia Vinerease in size, deeper in color | Contractions of — | Return to norimal | minora previa portion " Vagina] Transdave,dangaton Contractions in| Congeson dapper, acute Tower thd frmesominl pol in per to thi “Gers [Roem | Gansacions | contacion case and eras | onence _| Se Papa aA Sipe atceatan pote 8, Paraphilias 1, Basdophii: sexual urges toward children. Most common sexual assault 2. Exhibitionim:recurent desire to expose gonial ostanger ‘3. Moysuriam: sexu pleasure from watching others who are naked, grooming, or having ses, Begins eal in chilhood 4 Sadism sexual pleasure derived from others pain 5, Masochism: sevual pleasure derive from being abused or dominated 6, Bushisms sexual focus on objects eg, shoes stockings 1 Transvestite fetishism: fantasies or sctaal dressing by heterosecual men in female clothes for seal arousal by Diferetiate from transsexual and homosexual . Gender identity et by 240 3 years of age A. Strongest determinants of gender identity ave parental asignment and culture, not biology 7. lterisn male vubbing of genitals against ily clothed woman to achive orgasm: subways and buses {8 Zoaphili: animals prefered in sexual fntasis or practices 9, Coprophila combining sex and defecation 10, Lrophila combining sex and urination 11 Nectophila: preferred sex with cadavers 12 spony: ltered state of consciousness secondary to hypoxia while experiencing ‘orgasm, Ancoerccasphysaton, poppers. amy nitrate ntsc oxide USMLE Step 1: Behavioral Sciences ‘Table 5-3. Gender Identity and Preferred Sexual Partner of a Biolog Male ‘Common Label Gender Wentity | Preferred Sexual | Partner | Freteroseall ‘Male Female (Transvestite lets Male [ Gender identity disorder (was Female | Homosexual Male Sexual Dysfunctions 1, Pharmacologic effects ‘. Neutoleptia: may lad to erectile dysfunction du to dopamine blockage b, Dopamine agonist: enhanced erection and libido Trazodone (alphacl blocker and 5-H} priapism (prolonged erection in the absence of sexual stiulaton) Alpha-I Blockers may lad to impaired ejaculation 4, Retacblockers and tricyclics: may lead to erectile dysfunction «SSRIs: may inhibit orgasm in 15 co 20% of patent, retarded ejaculation 2. Sexual desie disorders 2, Hypoactive:defcienoy or absence of fantasies or desires, 20% of population, ‘mate common in women. Reasons low testosterone in men, CNS depressants, common postsugery, depression, maital discoed. Oral contraceptives depress Tiida, by, Sexual aversion; aversion to all sexual contact 3. Sexual arousal disorders 4 Hepa soul aro i. May occu in as high as 33% of females ii. May be hormonally related: many women report peak sexual desire just riot to menses ii, Anthistamine and anticholinergic medications cause decrease in vag 1a lubriction| >, Mate erectile disorder (impotence) i. Primary: never able to achiew erection fi. Secondary: was able once to achieve erection + 1040 20% lifeime prevalence, point prevalence is 3% + 50% of men treated for sexual disorders have this complaint + Incidence 8% young adult, 75% of men older than 80 + 50% more likely in smokers + Be sure to check alcohol usage, dbetes Ii Assessments postage stamp test, map gauge iv, Treatment sildenafil (Viagra) and similar drugs, mechanical devices Human Sexuality 4, Orgasm disorders ‘, Anozgasmia (inhibited female orgasm) |, Clitoral and vaginal ogatm are physiologically the same i, 596 of married women older than 35 have never achieved orgasm, iii, Overall prevalence from al causes: 30% i. Likelihood to have orgasm increases with age ‘Inhibits. male orgasm (retarded escuation) 4. Usually restricted o inability to ongasm in the vagina ii 5% general prevalence ii, Difierentinte from retrograde ejaculation Premature sjaculation i. Mae gully ejaculates before or immediately afer entering vagina ii. More common if erly sexual experonces were in back seat of ear ot with prostitute i, Treatment: stop and go technique, squeeze technique, SSRIs 5. Sexual pain disorders 4 Dysparcunis: recurrent and persistent pain before, during or after intercourse in either man or woman, i. More common in women ii. 30% of surgical procedures of female genital ara result in temporary condition {iL Chronic pelvic pain isa common complaint of women who have been raped or sexually abused ', Viginismus: involuntary muscle constriction ofthe outer third ofthe vagina i, Prevents penile insertion fi Treatment: relaxation, Hegar dilators D. Masturbation 1. Normal activity from infancy to ofl age 2, Reasons 4. In adults lonely tive, bored relieve stress, help sleep by In children: normal, fees ood 3, Frequency: 3-4 times weelly for adolescents, 1-2 times weekly for adults, once a month for married people 4, Equally common in men and women 5, Abnormal ony if interferes with sexual or occupational functioning 6, Can lead to premature ejaculation in males who use it primarlly to reduce tension E. Homosexuality 1. 410 10%6 ofall males, 110 396 of al ferales 2 Issue is partner preference, nt behavior 4 Behavioral patterns of homosexual te as varie as those of heterosexuals iméitical 6& USMLE Step 1: Behavioral Sciences », Same level and variations in sexuality as for heterosexuals Varlety of ongoing relationship pattems Male-male relationships ar ess stable than are female female relationships (Over 50% of homosexuals have children Distinguish berwcen ego-syntonic and ego-dystonic homosexuality a, Ego-syntonic: agrees with sense of self person is comfortable bs, Ego-dystonic: disagrees with sense of elf, makes person uncomfortable «. Wego-dystonie: sexual orientation distress. NOT considered pathologic unless ego-dystonie 6, Increasing evidence of biologic contribution. Higher concordance rates for MZ twins (6286) than for DZ (22%) 7. No fference inthe sexual response cycle 1, Preference well established by adolescence 2. Felings of preference emerge 3 or more years before frst encounter Describe duration of fetngs with “As long as can remember” 9, Similar numberof heterosexual experiences reported in childhood and adolescence 1, Report experiences as “ungratifying” , 300 40% of al people report a least one same-gender sexual experience 10. Very feminine (sissy) boys 2 Pefor paying with dol take female role when plying house , Mach greater chance of being homosexual vee Most homosexual men were not feminine boys, 11. Very masculine girs only slightly greater chance of being lesbian F, Sexuality and Aging 64 iiedical 1, Sexual intrest does not decine sgoticanty with aging 2. Continued sexual activity means sexual activity ean continue (use it lose i). 3. Hest predictor of sexual activity inthe ekleryis availability ofa partner 4, For afer MI, sexual position that puts east strain on the hear: partner ontop 5. Changes in men: 2 Slower erection by. Longer refractory period «. More stimulation needed 6, Changes in women: Vaginal dryness by Vaginal thinning «, Can be reduced by estrogen replacement ‘Human Sexuality ‘Chapter Summary Soul Behar inthe Unted Sates The vast majority of ales and ferales have had sexy the age of 9 7m of teenage bith are to ued mater. These pregracis cause heath end soil problems for | rater and child 2H of teenage havea sealywansmited disease Feral ages: Highs STO incidence ate ran papi vs Hignest prevalence rte hespessmplesype2 Most cormoniyrepacted STDs: chlamydia for eral and gonortea for males Parapis Defntions and assocatons fox peopl, exbinés,voyeursm, ads, masochism ftshism froteuism, zoophilia, coprophilia, opie, necrophilia, and poh Review lable 55 dingushng among: blerosen, tenses, assem and homosenl Sexual Dysfunctions | Reve the semaly reed side tes of nels, dopamine ago, rezone, bea biockrs, | try, and SSR. Review the descion and dterenal anong seul dese, em usa orgasm, and sexs pin dsorders Aasutaz somal fo os, Orb conse adamant wth caso | social acy, Homasenslty is pat of norma sens response pattem, Facts and mys rearing homoseny are cused. ‘Sexy and gn nterest in sex cosines thoughout ie Useitorlose it” + Desi remans bu arousal Becomes more fica USMLE Step 1: Behavioral Sciences 66 imédical Review Questions 133, Suicide has increased incidence ina wide range of psychiatric disorders. In others, the association is closer to that ofthe general poplation. The suicide rate fr which of the Following disorders is most likely to be closest to that of the geneal population? (A) schizophrenia (8) alcoholism (©) schizoid personality disorder (D) majoratfectve disorders () borderline personality disorder 34. A young couple comes to you for counseling soon ater they are married. They say they have read that marital satisfaction changes over the course of the marriage and want to know what they should expeet over the course of ther ovn marriage. The physician tls ‘them that, although their personal experience may be different, overall marital satistac- tion tend 0 (A) increase with length of time marred (8) decrease with lenge of time marcied (©) increase gradually, reaching a igh point when children are in adolescence, then decline rapidly (D) decrease gradually during the childbearing years, then increases afterall children Ihave left home (2) increase during the preschool years, decreasing during grammar school, then rising again during adoleteence 38. A39-yearolddivored Hispanie woman presents with lethargy and fatigue. When ques- tioned she complains of difuse physical aches, though her health appears to be genealy 00d, She confeses that she ins hers eying “for no reason” She epots not ely fel- Ing sxing ony of her unl rir nd as ficult leeping especially waking upeaty inthe morning. Shes gven 3 pelninary diagnos of un-polar depreson. In adion ‘0 this diagnosis the strongest sk factor fr saicdein this patent would be the patient's (a) age (B) gender (© marital tas (D) overall health () ist to the physician A) fatigue 36. Your schedule indicates that you have an intial eppointment with a patient who i 50- Year white male. Flowing th amination the mow Hy aes agro or (A) gistointesinal problems (8) upper respiratory distress (C) esential hypertension {D) obesity (©) urinary problems According to surveys by the Centers for Disease Conte and Prevention, as of 2000, the ‘mest common health problem inthe United States is () cancer (B) heart disease (©) substance abuse (D) obesity 1B) dental caries A 32-year-old white woman appess at your office for ber annual physical exam. The physical exam shows the patient tobe i good health, although she i slighty overweight fad hes moderstely elevated blood presse. If the patient were to die at some point in the next 10 years, the most likely cause of death would be (A) unintended injaries (8) neoplasms (©) hear disease (D) homicide () aps “Mr. Jones has been complaining ofa depresied mood for several months His wif informs you that he tied il himself lst month wien he reported hearing voices tht told him to kil himself, He was hosptalied for 21 days and given a diagnosis of a major psychiatric disordez, When questioned, he reported having "given up” Upon further questioning, yoo learn that he has a 10-year history of alcoholism, Which ofthe following would pose the ratest risk for future completed suicide? (A) feelings of helplessness (8) maria status (C) affective disorders (D) past suicide attempt (2) schizopheenia Daring a 1yesr period, a physician practicing modicine inthe United States would be ict likely to encounter & patent suffering from which of the following, mandatory reportable diseases! (A) Hepatitis A (B) Lyme disease (©) HaIViaips (D) chicken pox (©) syphilis tiédical 67 USMILE Step 1: Behavioral Sciences 68 ihedical 41 4, ‘An earthquake recently devastated town in Northern Caifornis, Hlecticiy was shut of for several days, and many ofthe people in the area were homeless The mos Iikly pattern ‘of response ofthe affected population would be (A) widespread emotional aerefects that are usually mild snd transitory (8) disintegration of socil organization {C) incidence of post-traumatie stress syndeome in close to 2086 of those elected (D) children adapt wo the new ciccumstances moze quickly than do adults (8) increased divorce in the following six months You have been appointed 1 provide an asesment of the general eslth status of your local community. You have a limited budget and must therfore, focus on the most ike ly determinant of community health satus. Based on this information, your assessment should focus on (A) hospital be:popalation 1 (8) infant morality ate Pysiianpatint ratio (D) general montality rate (2) quality ofthe physical and soil environment Following surgery for the removal af her appendix, «female patient comes to see you complaining of alack of intrestin seal contact with her husband. "We have ben fight- ing so much lately” she says. “Between that and the pressure I feel at work, just don't ‘know what todo anymore” Medical history shows tht sbe hasbeen taking diazepam for ‘the past two years and orl contraceptives forthe pat five year: Which ofthe following can be safely excluded a unlikely to result n the ported suppression of libido? (A) oral contraceptives (B) marital discord (©) postoperative recovery (D) work stress (2) diazepam At the conclusion of her annual gynecological exam, a 34-year-old mactied Hispanie woman confides to her physician that her interest in sex hasbeen “spot lay. although she has sexual relations with her busband at least once a week, she reports only feling ealypossionate”in the week just prior tthe onset of menses. She reais fom sexual intercourse during menses, The wornan wants to know whats wrong with he. The phys cian’ best response would be (A) “What medications are you taking” (B) “Is your husband sometimes abusive?” (©) “This 2 normal pattern of sexual arousal reported by many women” (D) “You may find that an erotic move wil simulate your sexual desire at times when you do not fel pasonate” (E) “How often does your husband want to have sex?” (B) "Tim going to have you tlk to2 friend of mine who specializes in this srt of thing” (G) "You might consider abstaining from sex fora while until you fee mote sure that you ig 8 . _Review Questions A woman reports to her physician that she can only achieve orgasm when recalling pre ‘ious, abusive boyfriend Sospecting the presence of paraphilia, the physician should explore for further evidence of (4) coprophilia. @ (©) sadism (D) frotteurism (8) voyeurism () exhibitionism {G) pedophilia (1 fesishism (masochism 0) roophila (6) necrophitia The medica record of «65-year-old white male details along list of medical conditions, including diabetes, gastric ler, recurrent headaches, and peripheral neuropathies. In Audition, the record indicates thatthe patient has a history of substance abuse, although fo specifics ae provide. When interviewing the patient, the physician is most likely to tiscover that the substance abused by the patient most likely was (A) aleohol (B) cocaine (©) cafiine () ecstasy (@) hallucinogens (©) inhalants (© opiates (GH) dative ypmoties [A 21-year-old male patient is brought to the emergency department by his parents who are concerned because he was stumbling around their hous, waving his arms inthe ai {and would not respond verbally to their questions. When examined, the patent appears ‘nxous, ith eleveted heart rate and clammy skin. A slight tremor is evident inhi hands fand his pupils are diated. Over time, he becomes verbal and reports that he elt ike he ‘vas loating ot of his body and that words spoken to him seemed lke insects that should be swatted away. He also admits to having recently taken an ilegal substance. ‘The patients behavior and physiology are most consitent wit intoxication due to (A) cocaine (B) inhaled paint thinner (©) marijuana (©) mescaline () phencjlidine (8) phenobarbital imeédical 69 USMLE Step 1: Behavioral Sciences 48. *. ‘A 72-year-old marzied man whois being treated for elevated cholestrol ask his phys ‘an about normal sexual function in the elderly. Ar this pont, the physician should inform the patent that (A) loss of interest in exis a natural part of aging (8) although men maintain interes in sexual atvity, women lose interest as (©) more mental and physical stimulation may be required to achieve erection (©) sexual activity should be limited to once a month to reduce cardiovascular stress (&) he willbe provided with a prescription for an anti-impotence drug so that itisavall- able when he needs it cy age The police bring a 22-year-old white male tothe emergeney department, From the out= set, he is belligerent, agressive, and violent, requiring the efforts of several officers to restrain him. When questioned the patents paranoid. Physical exam shows him to have muscle rigidity and pupils that move up and down rapidly. The patient had previously ‘been trated for opiate overdose. What neurochemical mechanisms aze most likely account forthe patient's curren behavior? (A) reduction in levels of GABA (B) antagonism of the glutamate reeptors ) partial agonist ofthe postsynaptic serotonin receptors (D) antagonism of the locus cerelose pathway and blocking of substance P (B) increases in GABA and inhibitory protein His parents, who ere concerned because their 17-year-old son is “ost not himself” bring him into the emergency department. Peliminary examination shows the boy to be drowsy, with lured spech, pupillary constcton, lethargy, and generally positive affect Based on this inital presentation, the boy is mos likely intoxicated with (A) caffeine (B) cannabis (©) cocaine (D) Isp 4B) aleobol (F) inhalants (G) phencyctidine (89 nicotine (opiates (sedative bypnotcs Answers 8 Answer: C, All options presented have suicide rats higher than the general population, but schizoid personality disorder has the lowest associated rte. Aleoholism and depres sion=15%,schizophrenia=10%%,borderines=5 t9 7% Answers D. Marital satisfaction tends to be lower for couples with chien, and tose ‘shen the children eave home. Answer: C, sing over age 45, mle, in poor health and emerging from depression ae all, risk factors for suicide, but none ofthese are true for tis patient. The fact that she i divorced, and the social solaton that may bring, isthe strongest risk factor along side the diagnosed depression. Answer: C, Essential hypertension isthe most likly diagnosis resulting from an office visit by « male wo his physicians. Answer:E. The key here is the phrase “health problem” More people have dental cavities than anjting else listed [Answer:B. For males in the same age range. the leading cause of death is accidents ‘Answer: D. Past suicide attempt isthe strongest rik ator. Sense of hopelessness is second, “Answer: D, In order from mest to least iy: chicken pox, HIVIAIDS, syphilis salmone, hepatitis A, “Answer: A, The aftermath of natural disasters finds many people suffering from distress reactions These reactions, however, tend to be relatively mild and resolve themselves of ther own accord and generally fall under the diagnosis of acute tess disorder, After nat~ tral disasters there tends tobe sn increase in social organization. PTSD incidence i clos fr to4% (Mt St Helens). Adults adapt more quickly than chiléren, Divorce rates tend to decline inthe perio just afer a disaster. “Answers E. The quality ofthe overall environment isthe main issue. The other, technical sountlng, options ave ll indicators of community health, but are not the most important eterminant. Infant morality is one ofthe strongest predictors of ie expectancy, but not ‘of overall health ofthe comaunity ‘Answer: E. Although the tiers have been shown to suppres sexual desc, diazepam has Answer: C. Many women report peak sexual arousal just prior to the onset of menses ‘Reassure the woman that her experience is normal. she needs reassurance not problem solving or medical intervention “Answers. To qualify for masochism the semua act must be the result of gratifation that includes receiving pain in ether realty or fantasy. "Answer: A. Alcohol i the most abused drug for any age. Note thatthe patients symp~ toms with the exception of the headache, are all inked to long-term alcohol use. Answer: D. The patient presents behavior and symptoms of someone high om hallucino fens. Although cocaine may aso induce aniey, the case lacks the other cocaine-reated symptoms. Answer: C, Sexual interest does not decline with age for ether men or women. A pre: ‘scription should not be given without an identified problem, __ Review Questions LUSMLE Step 1: Behavioral Sciences 49. Answer:B, The presenting profiles most suggestive of PCP intoxication, which produces its behavioral effects by antagonism of the glutamate receptors and the activation of dopamine neurons. 50, Answer:I. Pupillary constriction ang lethargy are the key features her Learning and Behavior Modification in the behaviorist model of lenning and behavior modification, internal states, subjective impresions, and unconscious processes are not relevant. Al that matters i the objective data, i. nly what can be seen, observed, and measbred. The behaviorist definition of learning: & relatively permanent change in behavior, not du to fatigue, drugs, or maturation, “The two main types of learning paradigms are assic conditioning and instrumental condi tioning ‘A. Classic Conditioning ucs vor Uncondtoned Stmmuls >» Unconaitioned Response [sight of food) (calvaton) ca Conctioned Response os. (ealvaten) Conditioned Stimulus (bal) Figure 6-1, Classic (or Respondent or Paviovian) Condtioning 1. Incassic conditioning the conditioned responses elicited by the conditioned stim lus after repeated pairings ofthe UCS and Cs 1. The Pavlovian experiment paired the ringing of abel with the bringing of food so that eventually the sound ofthe bell elcted the salivatery response, which previously occurred only with the ight of the food, 1, Or. for example, patient receives chemotherapy (UCS), which induces nausea (UCR, Eventually the sights and sounds of the hospital alone (CS) elicit nausea (now aC), 2. A now stimulus elicits the sme behavior, Note thatthe triggering stimulus (CS) ‘occurs before the response 5, Stimulus generalization: an organism's tendency to respond toa sini sim with the similar response. If slivation response had been conditioned to atone of 1,000 ‘CPS, an 800 CPS tone will iit simile response. O, in the second example, gener lization willhave occurred i any hospital,or even meeting physician, comes to elic- itmauscs from the patient. 4. Extinction: ater farming has occurred, removal of the pairing between the UCS and the CS results in a decressed probability thatthe conditioned response will be made The USMLE Requires You to Know + Dierences between css and operant caondtoing + Tipe of renorerent + The eles of ferent reinforcement schedules plications of behavior pnp in nical stations + Te behavioral substrate of deren + The aunnesrelatonstip beween arcs and performance chavo approaches to pan management inedical 75 USMLE Step 1: Behavioral Sciences For example, breaking the pairing between chemotherapy and the medial sting by ving chemotherapy at home. The nausca-lciting properties of hospitals will be extinguished B. Operant or Instrumental Conditioning, 1. In operant conditioning, a new response is emitted, perhaps randomly at fst, which results in a consequence, 1. The consequence acts as reinforcement and changes the probability of the responses future ozcucence b. In the Skinncr experiment, pressing a lever resulting inthe delivery of food After receiving food, the bar pressing behavior incressed, Because it changed behavior the food is a reinforcing event 2. A new response occurs to an old stimu. Note the ment) occurs afer the response. igering stimulus (eingorce Figure 6-2. Operant or Instrumental Conditioning 5. Definition of rinforcement (operant paradigm): a ssiaorce is defined by it effects, ‘Any stimulus isa reinforcer iit increases the probability ofa response, 4. Types of reinforcers 2. A postive reinforcer isa stimulus that, when applied following an operant response, sirengiiens the probability ofthat response occurring, i. gs increased pay lads to increased work from an employee ji. Fg. increaced complaining ead to increased attention fom the ns ing sa A negative reinforce i » stimulus that, when removed following an operant "response strengthens the probability ofthat response occuring i. Figo a child learns that he can stop his parents’ nagging by cleaning up his toom. i Avesive stimuli suchas aloud noise, bright light, shock, can often be negative reinforcers Both positive and negative reinforcement lead to an increase in response fe quency or strength 4. Punishment, like negative conditioning, usually uses «noxious stimalus 1. However, this stimulus is imposed to weaken response. li. Ordinarily punishment should be paired with positive reinforcement for altemative behavior, 74 iiiedical Learning and Behavior Modification iii, Exgs physical punishment of a child will suppress naughty behavion, ‘bat may fade when th punishment is removed and may model aggres- sve physical behavior forthe child . Exsnctin refers tothe removal of renforcer, resulting in lower response fre ‘quency and strength i. ga nurse who is bombarded by constant complaints from a patient slops paying atention to the patent whenever he complains ii, £4gya chilis jgnored bythe parents when be throws temper tantrums if succes, the unwanted behavior wil top. Table 6-1 Types of Reinforcement STRY To Ada Remove Behavior (R) Stops Punishment Batinction Continues Positive reinfo | ‘Negative reinforcement 5 Reinforcement schedules ‘4 Continuous rinfarcement: every response is fllowed by reinforcement. i. Results in fst learning (acquisition) i Results in fat extinction when reinforcement i stopped >, Intermittent (or partial ceinforcement: not every response is reinforced i, Learning is slower ii, Response is harder to extinguish fi Exgy a chil crows 2 tantrum and the parents ignore it fr long pesi= ‘ods of time in the hope thatthe child will stop. They don't want to reinforce such behavior with attention. However, if their patience ‘wears thin and after along spell of ignoring, they attend tothe baby, they ae puting the child on an intermittent reinforcement schedule and will find it harder co extinguish the tantrums. ix, Extinction of intermittent reinforcement often requires a change back te continuous reinforcement. 1 Interval schedules: based on the pasage of time before reinforcement iegiven + Fined interval schedule reinforces the response that occurs after a fied period of time elapse. + Response pattern on and off + Animal or person learns to delay response unt near end of time od gs cramming before an exam or working extra hard before bonus atthe holidays + Varidble interval schedule delivers reinforcements after unpre- “icabl time periods elapse + Higher, steadier rate of responding LUSMLE Step 1: Behavioral Sciences + Cannot lear when next response will be tenforce, leading 19 a steadier response rte + Eig. pop quizzes or suprise bonuses st work vi. Ratio schedules: based on the number of behaviors elicited before reinforcement i given delivers reinforcement afer a fiaed number of + Produces high response rate + Rewards set of behaviors rather than a single behavior. paying workers on a piecework basis. + Variable ratio schedule delivers veinfooament afer ¢ changing ‘numberof responses + Produces the greatest resistence to extinction +g, in gambling large nuraber of responses may be made without ‘reward. Since any response may be the lucky ope, peson keeps on ‘eying. Sot machines “Table 6-2, Reinforcement Schedules Co ~~ Gontingency Time Behaviors Schedule: Constant | Fixed interval (FT) | Fined ratio (FR) | Changing Variable interval (VD) | Variable ratio (VR) | 6, Spantancous covery: aftr extinction, the response occurs again without any further reinforcement 7, Secondary reinforcement symbol ora token guns reinforcement valve because of its association with a real reinforcer (eg, money is not valuable in itsef but because of what you can do with it) . Modeling, Observational, or Socal Learning 1, Watching someone else get reinforcement s enough to change behavior 2. ollows the same principles asin operant conditioning 3. Correlating the effets of watching violence on television with committing violence “the real world” tems from this concep. 4 Part of why group therapy works 5. Other applications: assertiveness traning, socal skills traning, preparing children for ‘various frightening of painful medical or surgical procedures Learning and Behavior Modification _D. Behavior Therapy and Behavior Modification Focus on treating symptoms directly rather than changing undesying intenal conflicts 2, Therapylmodication based on classic conditioning 4. Systematic desensitization i. Ofen used to treat anxiety and phobias ii, Based on the counterconditioning or reciprocal inhibition of anxiety responses + Sup I Hierarchy of fea-elctng stimuli i created, building fom least to most stressful, cp % Therapist teaches she technique of muscle relazation, @ response tha is incompatible with anxiety. + Step 3: Patient is taught to relaxin the presence, el or imagined, of ‘ach stimulus on the hierarchy from least to most stressful li, When the person is related in the presence of the feared stimulus, objectively, there no moze phobia. ix. Note that this works by replacing anxiety with relaxation, an incom patible response , Exposure i. Simple phobias can sometimes be treated by forced exposure to the feared object. fi, Exposure maintained nti fear response is extinguished ii, Reg fear of heights treated by havin patient to ride up elevator ix, In more exteme form, called “flooding” or “implosion” theapy . Avsrnte conditioning occurs when a stimulus that prodoces deviant behavior is paired with an aversive stimulus, i. Key properties of the orignal stimulus are changed + Eg, Pavlov’ dog being presented with spoiled meat upon ringing bell The dog does aot slivate, but insted recoils as the spoiled meat is presented, li, Used in the treatment of kcoholism and some forms of sexual deviance + Exgaan aleoholie is given a nauses-inducing drug (disliam) when- ver he drinks so thet drinking eventually comes to oliit unpleasant ‘rather than pleasant events chil peppers and thumb-sucking 2. Therapy/modifcation based on operant conditioning 2 Shaping (r successive approximations) i, Achieves fnal target behavior by reinforcing successive approxima tions ofthe desired response fi, Reinforcements gradually modified to move behavior from the more general to the specific responses desiced. ii, Eg, am auitc boy who won't speak i ist reinforced, perhaps with ‘candy, for any utterance. From those utterances, che appropriate phonemes arc sclected and reinforced until the child utters the sought after sounds, Eventually, eeinorcement is contingent on his using speech correctly in the proper context. médical 77 LUSMLE Step 1: Behavioral Sciences ». Extinction 1, Discontining the reinforcement that is maintaining an undesired behavior Ti. Eg. if complaining results in a patient receiving alot of attention, stopping the attention will eventually stop the undesired behavior ii, Eg institating atime out” with children wo are acting inappropri ately or test-takers who are anxious Stimalus control i. Sometimes stimuli inadvertently acquire contol ever behavior. When this iste, removal of that stimulus can extinguish the sponse 5. Ea persons eating behavior i ted toa particular stimula, such ax television watching. Reducing the time watching television should reds tae amount eaten, si Egy an inseronic is permitted in his bed only when hes so tied that Doe flls asleep almost at once, A. Biofeedback (neurofeedback) i. Using external feedback to modify internal physiologic sats, Ji Used to be thought that certain functions ofthe autonomic nervous system (heart rate, blood pressure, body temperature) were beyond the Selberate contol of a person. We now know that both animals and humans ean atin « measure of control over some of thai own bolily Functions through the techique of biofeedback, * Often uses electronic devices to present physiologic information, eg heatt monitor to show heart rate ii, Biofeback involves providing the person with information about bis internal response to stimu and methods to control and/or modify them, J. Bfeedback works by means of trial-and-error larning and reuizes repeated practice to be effective, Uses treatment of hypertension, migraine and musde-contraction Ineadaches, Raynaud syndrome, torticolis, cardiac arthythmis, and ‘i. Galvanic skin response: reduced skin condtivity = ansiety ection vil Most biofeedback aft the parasympathetic system i. Gradually removing the reinforcement without the subject discerning the difference li. Eg, promoting smoking cessation by reducing the nicotine content of the cgarctes geaduall and “silently” over a period of time ii, Eg. gradually replacing postoperative painkiller with a placebo Learning and Behavior Modification E, Behavioral Models of Depression 1. Teacned helpless Laboratory model of depression All normal avoidance responses ae extinguished, ji. A ratis shocked and not allowed to escape. Eventually the rat will not take an obvious avoidance route even whea itis offered «Symptoms of helplessness (in animals) include possi, norepinepherine ‘depletion, digcuty lerning that responses that produce relief, weight, and sppetite loss <4. Characerize (in people) by an attiude of when nothing works, why bother" «Increased levels of GABA in hippocampus decreas likeihood of leaned help lessaes esponse 2. Lowrate ofsesponse-contingent reinforcement 1. Another explanation for depression by, Too lite predictable postive reinforcement Person may lack the social shill necessary to elicit this positive reinforcement. 4, Depression can be seen asa prolonged extinction schedule. Ress in passivity Special Tops 1, Preparedness and learning ‘. Certain neutral stimuli ate more likely to become conditioned simul than are others 'b, People easily develop dog phobias but few develop phobias of electrial outlets, ‘which can certainly be as dangerous a dogs. «Theres no euch thing a universal teinforcer. Reinforcement depends on the internal state ofthe organism. i. Exgsconsider how likly you are to say “thank you toa turkey sandwich offered when you missed lunch versus jue afer Thanksgiving dinner Good Poston Adjustment Low Medium Anxiety High Figure 6, Relationship Between Anxiety and Postoperative Adjustment 2, Curvilinear relationship between anxiety and learning «Too much or too litle anxiety has a disruptive effect on learning. by, This curvilinear relationship applies to ‘Anxiety and performance Fear induction and adherence (heath belief mode!) ‘Motivation and learning Stimulus complesty and personal preferences Postsurgical recovery and anxiety 2. Behavioral approaches to pain management 4 Involves no assumptions about physical or psychological argin of pain (or even if person really i in pain) ain is subjective state and, therefore, not objectively measurable Focus ison pain behaviors (complaining taking medications, misting work) rather than on subjective state or unconscious determinants o 4 Involves assessing changes in pain behavior and changing en ronmental con- tingencies, including medications ‘Time contingent: takes control and administers medication at preset intervals Contrasts with horpice Pain contingent approach Emphases self-control and sel-administered pain medication Learning and Behavior Modification Tothik ke behaviors, focus oly on what canbe see, observed, or messed Tho primary types of ering + Classi condtoing an od response oa new simul tigering suk must precede response + Operant conditioning new sponse oa ld stimulus: igeing simul (enforcement) must | ‘low response Types ot reircerent pte: psi, regave, purshmet, and extinction Contiouusrercement rakes kearring easier and extincion ease: Intermitent enforcement mates leaning barder and extinction ard 1)pes of itermitent enforcement fed inte, variable nea, ied rai, verb ao arable ato retorcemen is hardest o extinguish, vento ening sane pnp pe ming etna py igs | Gis conto iis | + Spent derstand | + Spree mining srs rte etigsites | + este condo cage te rg stint pr an ve eso | pct coining eis | + Shaping succesive appromatos rd desired behavor | + tender el enor reong apt. Teo? | + Sil costo: avin al pod unwanted behave | + bent andere conning change neal pg eer ick — | + fad gata ono ace geen we manag bear | | oder sus spiral for mos Iering performance, and medal ewe. ‘The behavioral approach opin management focuses on the behav produced by the pin, notte | pain te @ tar, 1 Defense Mechanisms ‘A. General sues 1, Defense mechanisms atea concept born out of Freudian psychology. Recall hat the Freudian payche consists of 1d: instinctive urges, sex, aggression, and other primary proceses by. go: ational an language-based executors linking to realty « Superego: the conscience, the moral compass insisting on socially acceptable bbhavior, sometimes to the point of individual deprivation 2, Defenses are the primary tols of the ego, used to manage the internal conflicts beween the demands ofthe id and the restrictions ofthe superego. They aze the means by which the ego wards off anxiety and controls instinctive urges and wnpleas- antalfects (emotion. 4 All defences are unconscious. 1, Defenses change overtime. «Defenses are adaptive aswell as pathologic. We all use defenst all the time, They ae how we cape 4, Psychopathology isan issue of intensity and exten. Psychopathology ~ too ‘much all at once or for to extended a period of time. The key isue in psycho pathology is the degree to which the use of defense mechanisms is dis ruptive of «person's ability to deal wth the world around him or her. Ualike behavior, defenses are dentifed by what the person does in conjunction with his or her internal (unconscious) thought processes B, Four Clusters of Defenses 1, Nateisstie defenses 2. Projection: pesson attributes his or er own wishes desires, thoughts or emo: tions to somecne cle, Internal sats are perceived as part of someone else or ofthe word in general i. Beamples: + A man who has committed adultery becomes convinced that his wife ie having an afc eventhough there is no evidence of it + Agi tals abot her doll shaving certain Feeings, which ae reall wit the pit feels + A physician believes thatthe autsng staffs uncomfortable talking to him, when in fac, he i uncomfortable taking with them fi, Paranoid delusions result from the use of projection. Note The USMLE Requires You to Know. + Hon to recognize the presence of defense seth andthe impact they have on hun behaviors and perception, + Special ttenton should be paid tothe impact these defense mechan may fae on medical praice * Which defense mechan «ae commonly scat with specie pachitc diagrses USMLE Step 1: Behavioral Sciences ', Denial: not allowing realty to penetrate. Asserting that some clear feature of| eternal reality just isnot tre, Used to avoid becoming awate of a painful aspect of reality i Examples + Ate surviving heart attack, a patent i lifestyle as iF nothing had happened + A.cild who is abused insists that she bas been reted well + A worman prepares dinner for her husband expecting him to come home, eventhough he died a month exci. 4, Often the first response to bad news, such as the impending death of a loved one o oneself Substance abusers ae often “in denial, aiming that they are not addicted and do not have a problem in the face of cleasly dysfunctional or dangerous behavior « Spliting: people and things inthe world are perceived a all bad or all good (God ofthe Devil). The world is pitured in extreme terms rather than a more realistic blend of good and bad quit, Examples “This doctor is a mizacle worker, but that doctr is totally incompe “He's just so perfect and wonderful” says atenage gil in love “No one from that family will ever amount to anything; they are all just plain no good” Borderline personality disorders ase splitting and vacate between secing individuals inthe world a all good or al ba, Prejudice and stereotypes ar often the ceslt of spliting. Tnnmature defenses ists continuing his Blocking: temporary or transient block in thinking, or an inability to remember i. Examples + Mr Jones, you are suffering from. goe I jast cant remember whet itiecaled” + A student is unable to recall the fact needed to answer the exam ‘question, slthougl he reals it ase walks ou of the exam. + In the middle of « conversation, a woman pauses, looks confused, and asks, "What was jst talking about” 4, Blocking is disuptive and can be embarrassing. 1, Regzesion: returning to anealir sage of development. “Acting childish’ or at least younger than type for that individual i. Examples + An older patient giggles uncontrollably or breaks down eying when told bad news, + A-busban speaks (ois wife in "baby tlk + A patient les in bed curled up ina fetal position. fi, Ply is regressive Lea more re, simpler expression from a eal age fil, Regression ie common when people are tied ill, or uncomfortable. ix, Enuresis that develops ina child who previously had been continent following the birth ofa new sibling ithe result of regression, Sia ‘hen a nev child is born oder chiléren who have been weaned may demand to go back to breast-feeding. . Somatization: poychic derivatives are converted into body symptoms. Feelings faze manifest as physical symptoms rather than psychological stress i. Examples + Getting a headache while aking an exam + Feeling queaty and nauseated before asking someone out ona date 4 ringing inthe eas while making a presentation for Grand Rounds {i Extreme forms of somatization are diagnosed as somatoform disor ders (see section on DSM IV). iii, Symptoms created are physically real, not merely imagined 4 Introsstion (entification): features of external world or persons are taken in and made pat of the self. The oppose of projection i. Brampless + Aresident dresses and acts like the attending physician, + Acdild scolds herself oxt loud in the same manner that her mother ‘scolded her the day before + Axcenager adopts the syle and mannerisms of a rock star ii. When identifying with others s dane consciously, iis labeled “imita- ill, The superego is formed, in part, by the Introjcton ofthe same gen der parent a8 resolution to the Oedipal ess. i, Intzojcton is why children act like their parents. "T always swore that [would treat my children diferendy yet there I was sayin the same ‘hing to-my childzen that my mother always used to say to mel” 1 Bing a sports fan oF soap opera fan javolvesintrojection. Vi. Paiens in psychotherapy gan a diferent (hopefully heathiee) sense of sel in pan, by intojecting their therapist. 3. Anety defenses |. Displacement changing the target of an emotion or drive, while the person ving the feeling remains the same 4. Bxampless + A-man vwho is sngey at is boss pounds on his desk rather than telling his boss what he realy thinks. + An attending physician scolds « resident who later expresses his anger by yelling at a medical student. + Armarried man who is sexually aroused by a woman he ‘meets goes home and makes love to his wife. ii. In faily therapy, one child in the family is often singled out and blamed for all the family’s problems, ic is treated asa scapegoat by others displacing their symptoms ont this child, iil, Displacement often “runs down hill”. from higher 1 lower in a power hierarchy, Jv, Phobias are the result of displacement. [ USMLE Step 1: Behavioral Sciences ', Bepusson: an idea or feling ie eliminated from consciousness. Not that the ‘content may ance have been known, but now has become inaccessible. i. Examples: + A child who was abused by her mother and was treated for the hse, now has no memory of any mistreatment by her mother. + Aman who survived 6 months in a concentration camp cannot recall anything about his life during that time period ii. You forget and then forget that you forgot Content usually not recoverable without some trauma or psychoanal- jy. ierentiate from denial in that the reality was once accepted, but is now discarded One of the most basic defense mechanisms Isolation of fs reaity is acepted, but without the expected human emotional response to that reality, Separation ofan ide fom the afer that accompanies it i Beamples + child who as been beaten discusses the beatings without any dis. play of emotions. + A physician informs 3 patient of bis poor prognosis in bland, mat- terofact tones, + A patent who has had a finger severed in an accident describes the incident to his physician without any emotional eaction. fi, Fact without feelings fi, ‘The bland affect of schizophrenics, la belle indifference, that often accompanies conversion disorder is a manifestation of this defense ‘mechanism 4. Intellestualiation: affect is stripped away and replaced by an excesive use of| intellectual processes, Cognition replaces alect. The intellectual contents aca 100 = 130 ve Note that as CA goes up, if MA stays constant, 1Q goes doven Psychologic Health and Testing , Deviation from nozms method i. For each age range (cohort), take a sample ofthe 1Q test scores ii. Convert this distribution of scores using a Z-scoe type formula to @ distribution where the mean is 100 and the tandaed deviation is 15 This provides a separate distribution foreach age fi, Any test score for any person at any age can then be converted to ‘nth this distribution to provide a standardized comparison. These precomputed comparison distributions come as charts accompanying ‘the [Q test soring kit fi. Ifa child age 10 scores 25 on the test, simply find the table forage 10 and lok up a score of 25 to se what IQ level the score corresponds to ‘"Thesame sbeolute performance at diferent ages gies diferent 1Q scar ‘nseatse diferent norms are used foreach age «Error margin for both mental age method and deviation from norms method is-85 points 6, IQs ‘4 IQs highly corelated with education and isan excellent predictor of academe schievement. 1, Mental lines ie dstributed across all ranges of intelligence although measured 10 maybe lower when assessed because of interference of symptoms. € 10s not elated to suicide risk, A. Autistic children tend to be of blow average intelligence, with 80% having 10s Tes than 70. Longitudinal tests or intelligence show: 4. Venplitle decline in the elderly ii. Verbal bility holds up best li, Pexceptual and motor tess shove some decline Cross-sectional studies of 1Q show lower level for older persons because older persons tend ta be less educated than are the young. IQs very stable from age 5 onward. fh, Increased exposure to verbal havior early in life leads toa higher 1. 1Q tess contain elements of cularal bias, asking about words and objects more ‘wile les familiar or nonexistent in others. Samia in some cul 7, Commonly used IQ tests 1. Wechsler Adult Intelligence Seale, Revised (WAIS-R) is for adults aged 17 and older 'b, Wechsler Inteligence Scale for Children, Revised (WISC-R) i for children aged 6017, «¢ Wechsler Preschool and Primary Scale of inteligence (WPPSI is for children aged 4106. 4. Stanford-Binet Scale isthe fst formal 1Q test (1905) and is used for eileen aged 2 18 Today it's most vsefl with children younger than 6 the impaired, (OF the very bright LUSMLE Step 1: Behavioral Sciences D. Personality Tests 1. Objective test: simple stimuli (usually questions), estected range of esponses pos sible (slectetween choices given, scored mechanistically using seoring key: no cin- seal experience required to score. There are two types of objective personality tests 4 Citerion referenced i. Results are given meaning by comparing them with a preset stander, ii. Eg, USMLE Steps 1, 2.and3, ii, “Bvery student who scores above 75% wil pass” . Norm referenced i. Results at given meaning by comparing them with «nor ii High oF low scores indicate deviation from that group, i. “The top 2596 of the students will pas. Fv. Classic example: Minnesota Multihasic Personality Inventory (MMI) revised 1989 + $50 statements to which respondent answers true or false + Most widely use (and misused) personality test. Servesas eriterion for newly developed tests + Rav scores converted to T-ores with a mean of $0 anda standard , Neonates can discriminate between language and nonlanguage simul. «Infants do not learn language but learn to use the language capacity they are ‘born with (Broca' area). [At just 1 week ol the infant responde difeenty to the smell of the mother com: pared with the father. ae The MLE egret ow Capt od ples Sans — mere en fern sean fog meer — corer hi he sc issin peony cpa oni aati done age “eset optic eco +o ng i dep wah + Gander ptensl phaser Hon din cen opi tet me +g er Sere ares cua eh tes + Deng wih bce usu, 105 USMLE Step 1: Behavioral Sciences 6, Smiling 1. The smile develop from an innate reflex present at birth (endogenous smile) '. Am infant shows exogenous smiling in response to a face at 8 weeks, cA preferential social smile, eg. 10 the mother’s rather than anothers fice, appears about 12 to 16 weeks 7, Physical development 2, Hands and feet are the first parts ofthe body to reach adult sie b, Motor development fllows set patterns 1. Gragp procedes release i, Palm up maneuvers occur before ps dowa maneuvers ii, Proximal to distal progression fi Ulnar to eadial progression «Capacity to copy shapes follows i alphabetical order: 5. Circle, cos, rectangle, square triangle Ji, The exception isa “diamond” which can generally not be reproduced until age 4, First words (10 months), then fis birthday, then first steps (13 months) rows Cont Anse cues fom cms /\ nde Figure 8-1. Figures Copied and Approximate Ages 8. Key developmental issues 4 Brain growth spurt: “critical period” of great vulnerability to envizonmental influence i, Eetendng from last trimester of pregnancy through fst 14 postnatal months alist memories, roughly ages 2-4 First words (10 month), then fie birthday, then fs steps (13 months) i, Size of cortical cells and complesity of ell interconnections undergo their most rapid increase, Brain adapts structure to match envioa- mental stimulation, 4. Stranger anxiety: distress in the presence of unfamiliar people i. Ttappears at 6 months, reaches its peak at 8 months, disappears ater 12 mmonths ii, Can oceur even when child held by parent Separation anxiety: distress of infant following separation from a caretaker i. Appears at &-12 months ii Begins to disappear at 20-24 months fii, Continued separation, expecially prot 10 12 months leads to with drawal and isk of anaciti depression jx School phobia (Separation Anxiety Disorders failure to resolve sep- aration anxiety. Treatment focuses on childs interaction with parents not on activities in school {. Imprinting an interesting facet of attachment behavior in animals i. Some animals (geese, ducks, qui) will fllow the fist objec they see after beth fi, May even cun ti, sather than tothe mother, when fightened fi, Does not apply to humans Child Development inedical TT OO oovwvWtrlOO (0d p24 porn) 200) 9400 dog + | sa0015 - S08 seuiomy. | pay ‘sui jeanne annag (om 96) 24 sue ge ety (ouy2) eqn ms (ou) ‘poida wo spans zat _npvandde pop 280 + USMLE Step 1: Behavioral Sciences er ere a mart }~ g cate) ceunagem) arama) amcimnemal awl a OT ~ ~ ‘souoisop wwoMUdOPAAC PIED “1-6 914 g Child Development | stiysoone sn | soto vonesanacwe «| saty | womdopr9q 201098 ty | USMLE Step 1: Behavioral Sciences . Discipline of Chiléren 1. Be sure discipline s developmentally age-appropriate. Abstract, cognitive reasonings mean little oa child younger than 8 years, 2 If trying to stop a young child from biting another, don’t expect the child ro under stand how the other fel 3 Best application of discipline would be "time out 4 Punishment by hiting the childs too confusing; you are doing exactly what you are telling the child not to do 5, Discipline shouldbe clearly connected (in time and space) to behavior to be modi- fied. D. Teenagers 1. Identity formation isthe key issue. Issues of independence and self-definition pre- dominate, 2, The teenage years may be stressful but are not generally filled withthe type of trau- mas often portrayed inthe popular pros 3. Teenagers’ values reflec those oftheir parents. 4. Rebellion is manifested as minor disagreements regarding hai, music, res rend 5. Rebelion is most likely in easly teenage yen 6. Sexual experimentation with opposite-and same-sex partners is common. , Attachment and Lose 1, In cildhood 1. Bovelby postulates thee phases of response to prolonged separation of cildeen ged 7 months to 5 years, i. Protest: crying slarm, aggression {i Despair: hopes of regsining loved one fades Sil, Detachment: feelings of yearning and anger ace repressed », Paychological upset is more easly reversed in tages of protest or despair than after detachment ha tn Bocause separation has behavioral consequences, pediatric hospitalization must take it imo account through provision of parental contact (eg, roomingin practices flexible visting hours, assurances that mother wil be present when hil avses From surgery). 2, Inadals 4 Adults who are bereaved or are mourning the loss of 2 loved one aio go ‘through a series of phases, by. Intial phase (protest, acute disbelief) 4. Lasts several weeks li. Weeping fil, Hosiiny and protest «Intermediate phase (grief, disorganization) i 3 weeks to L year Fi. Sadness, yearning, somati symptoms Si, Obsessonal review, searching for deceased J May belive they see or hear deceased Conftonting reality 4. Recovery (or reorganization) phase 4. Reinvestment of energies and interests ii. Begins scond year Table 9-2. Normal Grief versus Depression ‘Normal Grief - fcr death, memories fade in intensity [Depression Normal up to 1 year “After I year sooner if ymptoms severe © Crying, decreased libido, weight los, Tonging, wish to se loved one, may think they hear or ee loved one ina crowd Loss of other ‘Sica ide ‘Seilited, usually es than 6 onthe ation bare “Antidepressants not help Dealing With Dying Patients 1, Stages of adjustment (Kubler-Ros) 4. Denial b. Anger Bargaining 1. Depression ce Acceptance Same “Almonmnal overideniGeation, personality change Tow of elf ‘Suicidal ideation i common ‘Symptoms do not stop (may persist for years) “Antidepressants helpful ) — | 2, People move back and forth through the stages. Not everyone passes through all stages or reaches adequate adjustment. 5, Similar stages for dealing with loss 4, Rules for dealing withthe dying: a. Tell the patient everything, Do not give false hope Keep involved in activities, or separation b «Allow person to talk about feelings 4 Avoid social ioltion. Child Development Hiédical USMILE Step 1: Behavioral Sciences 6. Childeen’s Conceptions of Hness and Death 1. Children do not see the seal world, do not live in the same world that we do, 1, Theyhawea limited cognitive repertoire; their thinking is concrete an egocentric, 1b, When they become i they may interpret this sa punishment andl may have all sorts of misconceptions about what i wrong with them. 2. Children rom birth wo 5 years old relly heve no conception of death asan irreversible proces, More then death, the preschool child is more likey to fear: ‘Separation from parents i, Punishment . Mutilation (Feud castration anxiety) 4. Only after age 8 or 9s there understanding ofthe universality, inevitability, and ee verity of death H, Facts About the Elderly and Aging 1. US. population aged 68 or older: 2, 4% in 1960 15% in 2000 25% in 2030 (estimate) Fastest growing age cobort is person older than 85 ‘The elderiy account for more than one-third of all eslth care expenditures Roughly 70% of men older than 75 are marred, but only 229 of women, 1395 ofthe elderly ae below the poverty line 8 This isthe same rat asthe rate for she total population 1b. The rte is two times greater for Hispanics and three times greater fr Blacks 5 0 10% of those older than 65 have moderate or severe dementia, 1 Okder than 85, the rate is 25%, a, 5086 of dementia cases are due 1 Alzheimer. 6. On 7. With the exception of cognitive impaicment, the elderly have a lower incidence of all poychistric disorder compared with younger adults 48, The elderly in the United States ace generally not isolated or lonely, but may not receive the same respect asin other cultures. 4 60% have children and most have frequent contact with them. 'b. The family is stil the major social support system for the elderly in times of it~ «, Insttaionalzation is undertaken only a last resort. 8, 85% ofthe elderly have at least one chronic illness 44 50% have some limitation o their activites. b Only $46 are homebound. 10, Among noninstitutionalized, 60% call heir heath excellent of good, 20% fit (Child Development 11, Scie rates per 100,000: Ages Males Females 651074 304 65 too 423 35 5 and older 506 55 12, Currently about 2 million, or about 6% ofthe elderly population are institutionalize. 13, One in 10 persons aged 75 or older i in a nursing home; for 85 and olde, the ratio is Lins. 1. Preventive occupstional therapy (OT) programs offer clear advantages over “just keeping busy to reduce decline in mental and physical heath inthe elder 15, Best predictor of nursing home admission: fll and fll related injures. Exercise improves balance and reduces risk of fallin the elderly 16. Life expectancy (1995): White female = 79.6 White male = 73.3 Black femsle= 739 Black male = 649 1. Abuse 1. Child abuse 4. More than 6:000 children ae killed by parents or caretakers each year in the United Sats '. More than 3 milion annually ar reported abused, S0% ofthese are confirmed by investigation, «Likely that many abuse cases unreported 4. Defining abuse i. Tissue damage ii. Neglect, fi, Sexual exploitation im Mental eruelty ce, Mandatory eportable offense up to age 18, i. Baile to do sos criminal offense i, tf case is eported in ers, the physician is protected from eal ability ii, Remember your duty to protect the child (separate from the parents, a well the duty 0 report. f. Clinical signs i, Broken bones infest year of life i, Sexually transmited disease (STD) in young children li, 9296 of injuries are sft tissue injuries bruises, burns, lscerations) Jn. have no physical signs Nonaccdental burn have a particularly poor prognosis. + They ae axzocated with death or foster home placement. + If burn ison arm and hands it was likely an accident + Ieburn is om arms bat not hands, i s more likely abuse vi. Shaken baby syndrome: ook for broken blood vessels in eyes inédical 11 LUSMIE Step 1: Behavioral Sciences Children at isk for abuse are i. Youngs than 1 year of ge fi Stepehitven i, Premature children im. Very ative Defective” children bh, Parents likely themselves to have been abused, andr persive child as ungrate ful and as cause oftheir problems |, Be careful not to mistake benign cultural practices such as “oining” or “most Dustion” as child abuse i, These and other folk medicine practices should usually be accepted. li. Key is whether practice causes enduring pain or longterm damage to hid i, Theat female circumcision as abuse im Look for an opening to discuss with patents how they treated child prior to seeing the physician 4. Children who are abused are more likely to: i. Be aggressive inthe classtoom ii, Pereive others as hostile ii. View aggression asa good way to solve problems jn. Mave abnormally high rate of withdrawal (gis) ‘ Beunpopula with school pers and other children; the feiends they do have ter! to be younger 2, Child sexual abuse 4. 150,000 to 200,000 cases of sexual abuse per year Homicide Rate eer 100,000 US Children Pecpetotor: MI Famty EE Acquaintance 2] stanger Figure #2. Relationship of Child Homicide Victims to Perpetrators 586 of sexual abuse cases ate within the family. 6 6096 of victims ane Female 75 REM sleep-low voltageandom, fast with sawtooth waves Figure 10-2. Sleep Architecture Diagram ‘Showing Stages of Sleep in Sequence WU Sleep and Sleep Disorders USMLE Step 1 Sciences B, Sleep Deprivation The cerebral cortex shows the greatest effect. ty to cope with one nigh sleep los “The west ofthe body seems relatively unaffected by sleep deprivation. Physical ett tion of the body comes fom the immobility that sa by-product of sep, not from sleep isl. ‘Only about one-third of sleep is made up. a. 80% of lost Stage is recovered ', About one-half ofthe missing REM is recover leep deprivation but bas the capaci- 1 longer the prior period of wakefulness, the more Stage 4 sleep increases during ‘theft part of the night and the more REMI decins. Short sleeper lot the later pat of REM slp, Iymphooyt levels decline snd cotisol levels rise in sleep-deprived individuals REM sleep appears to increase somewhat in both children und adult after Tearning, specially she learning of complex material, in the previous waking period. 4. REM seems to “lear out the brain” i. Removes partially processed material by “memory consolidation” ii, Dampens excitement, which ads in the prolonging of sleep. Adult human REM sleep is more fetalikethan is NREM sleep. REM provides cerebral stimulation while maintaining the slesping tte , REM sloop is essential to get the most out of studying, Its when most long term memories are consolidated by the hippocampus, REM deprivation ‘Does not impede the performance of simple tasks by, Imerfores with the performance of more complex asks Makes it more dificult wo leatn complex tasks 4. Decreases attention to details but not the capacity to deal with criss situations In bot adulte and children, during the fist 3 hours of sep a, There i a very large output of human grovth hormone (related to Stage 4 sleep). This is by fr the greatest release of this hormone in a 24-hour perio, ', Prolactin rises rapidly and dopamine fal in both sexes. «Serotonin (5-H) rises, 4. Sleep inhibits the release of thyrld-stimulating hormone (TSH) “Melatonin, which is not related to sleeping, bu to flings of lepine: 4. Produced in the pineal gland and directly in the retinas ofthe eyes by Sensitive to light via a pathway from the eyes «Release is inhibited by daylight, and, at nighttime, levels rise dramatically 4, Likely mechanism by which light and dark regulate cieadian rhythm «Responsible for “je lag” |. Delta sleep increases afer exercise and seems to be the result of eased cerebral tem Sleep and Sleep Disorders . Developmental Aspects of Sleep Hours ot Sleep 16 “4 woke 2 REM sleep 25% 10 118 days 6-23mo 2-3y 13By —BO-SDY BOY Figure 10-3. Changes in Daily Sloop Over the Life Cycle 1, Sleep develops during childhood and adolescence into adult patterns. bs “val Seep Time/24 Hours Neonate 16-18h ly 12h oy 10h ii6y ah USMLE Step jehavioral Sciences Age ‘Number of Sep Bernds/24 Hours Neonate «9 ray 2 soy 1 2 Infants ‘a. Premature infants do not demonstrat a discernible sleep-wake cycle. 'b. EEG demonstrates adultlike rhythms of slep and wakeflness by about 1 year «Neonatal sleep cyte starts a 30 to 40 minutes, gradually lengthens o 90 min- utes by tens 3. Adalts 4 Initial RENE cycee approximately 90 minutes, Subsequent eyes across the evening are shorter b. REM: 20% of sleep time Total sleep time/24-hour period decrease gradually with age 4. Elderly «a. Total sleep time continues to dectine ’. REM percentage remains constant (20%) up to around 80 years of age, then desing further. Stage 4, then Stage 3 NREM (delta sleep) vanish, Elerly often complain that they don't feel as rested as they used to fel D. Sleep Disorders 1. Natcolepsy a, The narcoleptic tetra i. Sleep atachs and excessive daytime sleepiness (E ik Catplxy(pathognomenic sign) iit Hypmagogchalacinaions (hypnopompic ean occur but nt pathog- nomonic) + Hypagogic while ling seep + Hypnopompic: while weking up ie. Sleep paras b. Narcoleey i disorder oF REM sleep: onset of REM within 10mninutes «Linked to dfceney in hypocein protein (resin transport 4 Treatment theres no cure) 4. The pathophysiology not wel understocd. fi CNS simulans control the EDS (metyphenidatdextoampbtaming) Sik Trcic anidepresants contol the cataplesy Ginipramine) ‘xy Modafinil (new}:a nonampetamine aerate to CNS simulans 2. Slep apnsasidromes the abseace of respiration during sep (beating elated steep disorder) 4 Coden Axi Step and Slep Disorders 1, Subtypes: 4. Obstructive (upper arway) sleep apnea + Rasping snoring + Overweight + Middle-aged fi, Cental (aphragmatc) sleep apnea + Eldely + Overweight + Cheyne-Stokes:60-second hyperventilation, fellowed by apnes ii, Mixed slep apnea Clinical presentation and features: i. High risk of sudden death during sleep, development of severe noc: tural hypoxemia, pulmonary and systemic hypertension (with ele vated diastolic pressure) fi, Nocturnal cardiac arshythmias (potemaly life-threatening) Wi. Bradyeardia, then tachycardia fi Males eutmumber females (premenopausal by 8 to 1 ‘EDS and insomnia often reported i. Heavy snoring with frequent pauses vii, Obesity is often part of the clinical picture, but not ahsays. vill. Short sleep duration, frequent waking, insomnia, decreased Stage 1 , Malingering: everything conscious i, Intentional syraptom production fi, Conscious motivation fi, Symptoms purely for secondary gain, eto avoid «court dat, mill= tary induction, or schoo! Is, “Treat by fostering relationship and understanding motive « Eastitious disorder: some conscious, some unconscious i, Intentional llness production ii, Unconscious motivation, therefore, compulsion ii, Patents aware of manufacturing their symptoms but unaware of why they go o such lengths ix. Both primary and secondary gain ‘% Munchausen syndrome or polysurgieal addition: chronic fattious ies with pial symptoms vi Manchausen by pron: inducing symptoms in others (eg, mother prodacing symptoms in er child) vii, Pactiiousdisarders require treatment: foster relationship and look for motive H, Post-Traumatic Stress Disorder 1. Manifestations: Re-esperience ofthe event as rcursent dreams or recollections (Aashbcks) Avoidance of associated simul Diminished responsiveness to external wold b 4. Sleep disruption or excess « Teriabiliy, 10s of controh, imprlsivity Headaches, inability to concentrate Repetition compulsion 2. Symptoms must be exhibited for more than 1 month; if less, diagnose as acute stress disorder 136 imédical Diagnostic and Statistical Manual IV (OSM IV) 53, Following psychologically stressful event outside the range of normal human experience {. Most commonly, serous threat to if family, children, home, or com nity |i, Common reaction to rape for women 4, Ofien long latency period eg, abuse as child, manifest symptoms a adult 5. Quicker onset correlates with better prognosis 6, Increased vulnerability i ‘Prior emotional varabiley, excessive autonomic reactions is prodicior of ', Prior cocainefopiate use 7, Adults recover quicker; very young and very old have harder time coping 8, Prevalence: 0.5% in men, 1.2% in women {9, Slop changes inves in REM latency; decrease in amount of REM and Stage sleep 10, Inreased trina secretion of norepinephrine 11. Treatment: group therapy to facilitate working through normal reactions bcked by disorder, seeking cathartic release SSRs can improve patients functional lve 1. Adjustment Disorder 1, Residual category use only if no other Axis applies 2, Ceteri: ‘4. Presence of identifiable stressor within 3 months of onset by, Symptoms lst les than 6 months afer end of stressor «. Symptoms ate clinically significant, with significant socal occupational andor ‘academic impairment 44. Nota gret response 1 Dissociative Disorders 1, Symptoms similar to patents with temporal lobe dysfunction 2, Reaction often precipitated by emotional criss 1. Primary gun: anxiety eduction Secondary gun: solution tothe crisis avoidance) 3, Subtypes a, Pogue 5, Sudden unexpected travel i Inability to reall one’s past, {ik Confasion of identity or new identity Amnesia 4. Inability to reall important personal information fi, Linked to trauma «Dissociative identity disorder (mutiple personality) 4. Presence of two of more distinct identities ii, Atleast two identities cotzol behavior ii, Inabilty to recall important personal information medical 157 USMLE Step 1: Behavioral Sciences 4. Depersonalization disorder i. Recurrent experiences of being detached from or outside of one'sbody ii Realy testing stays intact i, Causes significant impairment K, Personality Disorders 1. General characteristics 4 Infexbl, inability to adapt b. One way of responding «Lifelong 4. Allareas of life fe, Maladaptive response to stres: response to social context £. Can thrive if situation is ight Interactive interpersonal disorders 4. Patients rally annoy others fi. Symptoms are ego-syntonic lik Pationt seeks to change the world, nt self ‘Overall 10 to 158 prevalence (Nova Scotia and Manhattan studies) i. Concordance rate for MZ. twins three times that of DZ twins 2, Cluster edd or excentc—higher prevalence in biologic relatives of schzophen- ies higher prevalence in males a. Paranoid 1. Long-standing suspcionsness or mistrust of others: a baseline of mis: ii. Preoccupied with ses of trust i, Reluctant to confide in others jy. Reads hidden meaning inte comments or events 1 Carries grudges Vi. May have history of misdemeanor arrests vii, Differentiate from: + Paranoid schizophrenic has hallucinations and formal thought dis- orders paranoid personality disorder does not + Delusional disorder, paranoid type has fixed, focal delusions para- noid personality disorder does not , Sciaaid i, Lifelong pattern of socal withdrawal, and they lke it that way i, Seen by others as centri, isolated, withdrawn ii, Reseicted emotional expression i, Prevalence 75% of population 1 Mast times more than females | Diagnostic and Statistical Manual IV (DSM 1V) | samen ie ii Inability to maintain long-lasting relationships ii, Attention-seeking, constantly wanting the spotlight Seductive behavior b. Narcissistic i. Number of reported cases increasing fi, Grandiose sense of selfsimportance iii, Preoccupation with fantasies of unlimited wealth, power, love iv. Demands constant atention Fragile self-esteem, prone to depression vie Criticism met with inditference or rage vii Genuine surprise and anger when others dost do as they want, vii. Cam be charismatic Bordsaliae i. 1t0.2% of population ii. Females two times more than males ii, Very unstable alec, behavior sel image iv, In constant stat of esis, chaos ', Selédetrimental impulsivity: promiscuity, gambling, overeating, sub- stance abuse vi. Unstable but intense interpersonal relationships: very dependent and hostile, lovelate | vit Great problems with being lone | vii, Solématilation ‘x, History of sexual abuse x Common defenses: splitting, pasive-aggressive xi, Particlarly incapable of tolerating anxiety xii, Often coupled with mood disorder ill, 5% commit suicide inédical 159 USMLE Step 1: Behavioral Sciences 4. Antisocial 4,346 males, 1% females i Continual exieninal acts i, Inability to conform to socal norms: truancy, delinquency, theft, run ning away jw. Cant hold ob, no enduring attachments eches, aggressive Onset before age 15; if younger than 18, diagnose as conduct disorder ‘Three imes greater in areas of social disintegration; this matters more than docs SES vii, Comprises 75% of prison populations vii, ive times more common among relatives with disordec 4 ChustrC. anxious an fearful~behavors associated with ear and anxiety a. Avoidant i. Extreme sensitivity t rejection Hi. Sees elf as socaly inept ii, Excessive synes, high anxiety levels iu. Social isolation, but an intense, internal desire fr affection and accep 1. ‘Tends to stayin same job, sae ie situation, same relationships ‘Table 12-6. Personality Disorders e + — teat [Sa [scm Yas ne | . | Detintion | Arviores Twoated fete, | EscetrcBebav- | Bxesive emotion | Grandiose, over- | dence in fami- | dence in fami- | than women diagnosed in | | cee | Scans [ram oa Tipe fe | i gmes | set 160 iédical Diagnostic and Statistical Manual IV (DSM IV) bi, Obsesive-compulsive i, Onderlines, inflesble, perfectionist Ji, Mote common in males, fist bor, hard discipline upbringing ‘ik, Loves list, rules, order is, Unable to discard worn-out objects Doesn't want change vi Excesively stubborn vii, Lacks sense of humor ‘ii, Wants to keep routine ix. Diferentiate fom obsessive-compulsive anaety disorder. The anxiety disorder has obsessions and compulsions that ae fecal and acquired. Personality disorders are life-long and pervasive, © Dependent 4. Gets others to assume responsibility fi, Subosdinates own needs to others ii, Cant expres disagreement ‘Great fer of having to care for self 1% May be linked to abusive sponse Borderline Antisoci Avoidant Dependent Obsessive Compulsive Instability of | Does notrecog- | Shy ortimid, | Dependent sub- | Perfctionisic mood, self-image, | nize the rights of | fears rjoction | missive and inflexible, tnd relationships | others onder rigid 1. Women Prevalence: in | 1, Common | 1, Common |. Men greater sresterthan | men; 19 in 2 Possible 2. Women than women mea women eformingil- | greatertban | 2. Increased con 2 Increased ness ‘men cordance in mood disor Sdentical twins | ders in fariies | 1. Spliting 1 tolation | 2, Projective | Superego lacunae | Avoidance 2 Reaction fr ‘dentiiation| sation 3, Dissociation | 5 Undoing 4 Intllec tualization LUSMILE Step 1: Behavioral Sciences 162 iédical Chapter Summary | Thee DSM W ans of dagresis we: inal are, ~ pesonly dade and metal | exon phys eondins nd dead - phosocal and enone bls | | | ‘mgrtant dees wy dred in chided axe ert retardation, pease devopent sodas), and ater defi hpetaciiy dre ADH. | sticoperia a common ives pie ypychoss,ntegaton ofl thkig ed alos | | ably matin roma soa bebe Sibypes pond, cain, dorize, | etree, nd ves | | | | Postivesjmotons are what nama gesons donot hve but shizophrenis do. Neate syptors ‘ae what oral persons have bt shirophreis donot | | Amaltuncion ofthe dopamine newotrasmision system i responsible for stzophenc symptoms, | ts a ame ya Schieoptveria associated kunconal dec smooth pusut ee movements bac masing ‘evenielited potential aromas, nd reduction in preontal acl att inthe face of cogtve change. Mood Disorders: sthymia i onc depron; uniplr disorder (severe depression); | edttymia (il, chronic bpola:biplr dsr (manic-epression). Seasonal af dsrder (64D) schaacered by the onset of symptoms of depression during winter months abroad to depression: measurement of noepnephine metabolites, the | | cezamehasone suppression tt (OSD, andthe throtopirrdeang hormone (TRH) stmt test | | ing Disord: unio eros nd anor nena | | sy Dre: rea! ny or (AD) ps ese ope rte | (£c) and pic dso | sotom Ose sna ane, ponies psn ty dana. | | ext smattm dds, the patti arate of the mom soon oaton. | foc tacos dsr, te pater sna of ynpom prin, rt man. For _malingering, both symptom production and motwation are known. | | posctraumstc Sess Diode reexerence of event and anidance of assacited simul along wih | ether dystcionl symptoms foloning an event ouside the elm of normal human experience | Adjustment Osorder residual disorder only gen no thes As dagocss apps. Cannot be a aie respons | Disocive Disorders gue, arms, disci deity Src, and depersorazatondsorde | Pasonlity Disorders ong penaste gosyronic.Spctic ype: paranoid (aspicous, | | unsing, schizid (soled, witcoun) schizotypal (wei), ston (col and sect, racist (grandis) bodene (usable ih nese personel less), anol (iflong | ciiea, avian (xtra hp obsessie-compusve(oreoctupabn with ordetiness and | conto, and depercet et chest tte esponsy. Table Ms summaries he dares | estate peso rds. 9 9%. Review Questions A 6-year-ald boy was refered by his st grade teacher for evaluation ater she noticed ‘that he had tobe keeping up withthe tier children in his clas. After psychological testing and an evaluation interview, the boy's 10 is assessed at 62. Based on this informa- tion, thea hei an adult the boy most likely will (A) find work in sheltered workshop setting (B) have difculty with basic reading and math ills (C) lead anormal life with no special support required (D) require custodial care (2) will ned guidance for important life decisions ‘A45.year-od woman presents to her primary care physician complaining of fatigue and Ineadaches, Over the past month she reports that she has had tole sleeping difculty concentrating and episodes of crying for no zeason. In addition, she says that she feels sadand worthless, The neurological pathway most key implicated as the source ofthese symptoms is the (A) meso-limbie-ortco pathway (@) locus ceruleus pathway (©) nigrostital pathway (D) nvileus accumbens pathway ) glycolytic pathway 'A42-year-old woman has always been extremely neat and conscientious, skills she makes ‘ood use of asthe executive secretary tothe president ofa large corporation, Something ‘of perfectionist, se often stays long after normal working hours check on the punc- tuation and speling of leters that she prepared during the day. though hee work i impeccable, she has few close relationships with others. Her bos refered her for coun scling after repeatedly getting int fights With her coworkers. “They just dont take the job to heart” she sys disapprovingly about them. “Al they seem to want todo is joke around all day” The mos likely preliminary diagnosis for this patient is (A) obsessve-compulsive personality disorder (8) paranoid personality disorder (©) schizoid personality disorder (D) hysterical personality disorder (&) antisocial personality disorder (8) narcissistic personality disorder (G) borderline personality disorder (H) dependent personality disorder (avoidant personality disorder (D).sehionypal personality disorder Review Questions 168 inédical MLE Step 1: Behavioral Sciences. 95, A 22-yearold male patcat refuses to provide answers to standaed questions during an ‘nial history and physical exam, including eddress and telephone number. When asked the reason forthe refusal, he says he does see why the physician needs soc ireevant formation and eyes the physician suspiciously. When pressed farther, he asks angi, ok, are you going to teat me ot do have to get my lawyer?” This behavior is most consistent with 3 diagnosis of (A) obsessive-compulsive personality disorder (8) paranoid personality disorder (©) schizoid personality disorder (D) hysterical personality disorder (@) antisocial personality disorder (narcissistic personality dsorder (6) bordettne personality disorder (1H) dependent personality disorder (0) evoidant personality disorder {D)_schizotypal personality disorder 97. A patient’ past medical history indicates numerous admissions to a local public hospital, nosy ducing the winter months. On each accsion, he has been brought to te hospital afer passing out ina public locaton, His appearance is disheveled and his clothes are torn. He complains thatthe cold makes him dizey and asks the physician, with teas in his eyes, o “please help him get better" Neurological examination is unable to uncover any underlying cause for his teprted symptoms. The most likely preliminary diagnosis for this patient would be (4) ystayria (8) somatofoem disorder (©) delusional disorder (D) factions disorder (E) malingering 98. A man awakens to Find bright sunny day. As he dresses, he makes sure to put on a yellow ‘hit to ensue that it wil stay sunny al day This behavior can best be described a8 (A) delusion hallucination (©) ides of reference (D) magical thinking (©) ilusion 8. 100. to ‘Glen is a model patent on the ward. He never i any trouble. Whatever he stl to do Ihe docs. He sits quietly for hours, rarely talking, and hardly moving, except to ape the movements of those who pas by him. The most ike diagnosis for Glenn is (A) schizophrenia, disorganized type (8) schizophrenia, undiferentiated type (©) schizophrenia, paranoid type (D) schizophrenia, residual phase (E) schizophrenia, catatonic type According to twin studs, the strongest evidence of genetic ease i for (A) schizophrenia (@) bipolar disorder (€) unipolar depression (D) antisocial personality disorder () alcoholism [A patient sutfcring from paranoid schizophrenia has been on his mediation fora fll year. During this time, his postive symptoms have abated and he shows no signs of relapse. IFhe continues to be adherent with his medication, his chance of relapse over the ‘coming year is most likely to be 9) 80% (®) 50% © 40% (0) 30% ©) 10% ‘A.22-year-old mentally retarded male ives on his oven but works ina sheltered workshop setting. He hasan active soil life and is wel ike by his peers. He mets weekly with women in. Hypertension is predisposing factor | ‘15% ofall dementias inthe elderty i. Can often find some lteralizing neurologic signs «Biology i. Vascular disease is present, i. Affect smal- and medium sized cercbral vessels that infarct and pro- duce parenchymal lesions over wide areas of the brain 4 Treatment i, Thestment of underlying condition (hypertension, diabetes mellitus, Inyperlipidemia) fi, General measures for dementia. iiedical 171 LUSMLE Step 1: Behavioral Sciences 17 médical “ale 12 Dementia Aahsimer Type ess Vascular Dementia [Dementia albsimerype | Vascular Dementia Z| General deterioration Patchy deterioration | | More in women Mare in men Tater onset Bair onset | Mos common, 65% of dementias Less common, 15% of dementias pietecems [ Biology unknowe ology features hypertension [Progesve ome ick onset _ | tering nero sens i | No lateral signs 4, Bick disease 2. Affects frontal and temporal lobes b, Vary rire Similar picture to DAT 4. Prominent frontal lobe symptoms (personality change) «Reactive lio in frontaltemporal lobes f. CT or MRI sometimes shows frontal lobe invalvement but definitive diagnosis Iscnly at autopsy 5, Creutzldt-akob disease ‘Dementia caused by prion (ne DNA or RNA) bs, Rapidly progressive . Generally onset between age 40 and 50 4, Inily, vague somatic complaints and unspecified anxiety, followed by ataxia, choreoathetoss, and dysarthria ce, Fatal in 2 yeas (usually sooner) £. CT demonstrates atrophy in cortex/cerebellur & Notreatment 6. Huntington chorea ‘. Autosomal dominant by, Defect in chromosome 4 Males = females 4. Basal ganglia and caudate atrophy , Choreosthetod movements and dementia Onset between ages 80 and 40 1. Progressive deterioration bh, Dementia, later with paychoss progressing to infantile tate ath in 15-20 years Suicide is common, 7. Ragkinson disease 2. Decreased dopanin in substantia nigra 2. Anal preven fs 200 100,000 «Symptoms 4. Bradys il Resting tremor i, Piloting wemor iv, Maske facie 1 Coghe!rigty ¥ Shuing git 4. 40810 80% develop dementia «Depression i common treat with ndepressnts or electroconvlsve therapy «cn {Treatment T-dopa or depreny 8. Won disease 2, Defect in eromosome 13 1, Cerlopasnin deficiency «Abnormal copper metabolism 4. Kaser-lechor ings 9, Noval presse hoes 2 Symptom trad of i Demeatia it Urinary incontinence i. Gait praia magnet ait) Increased venides on CT «Norm presure on lambar puncture 4. Treat wth shun ispinal ap s"terapetie® . Hemispheric Dominance 1. Lefthemisphere ‘a. Language ' Dominant in 97% of population, 60 to 70% in left-handed persons «5 Caleulation-ype problem solving 4 Stroke damage to left is mor likely to lead to depression Larger in size than isthe right side and process information faster 2. Righthsmisphsce ‘a. Perception, artistic, visul-spatial bs, Activated fr intuition-type problem solving «Stroke damage to rghtf more likely to lead to apathy and initerence 4. Prosody resides her &. More 5-H receptors Organic Disorders USMLE Step 1: Behavioral Sciences . Aphasias 4. Lesion of fontal lobe (Brodmann area 44) 'b, Comprehension unimpained «Speech production is telegraphic and ungrammatcal 4. Often accompanied by depressive symptoms f & "T moves instead of “| went tothe movies" “Trouble repeating statements “Muscle weakness om the right side Note 2, Wernicke's Lesions of superior temporal gyrus (Brodmann area 22) Comprehension impaired mroveents eg. unable to Speech i Buent but incoherent fipa cin when asked o do “rouble repeating statements + Apis of bly i 4 * «Verbal paraphasias (substituting one word for another, or making up word) t & b, lean orto cry cut speci [No muscle weakness Resembles formal thought disorder “Manilike, rapid speech hyperactivity + Agosto recognize senscry sim 2g, al agnosia, unable to recognize abject when son but ble 8, Conduction terecopne when ous ‘Lesion in the parietal lobe or arcuate fasciculus + Alea acquired disorder of ', Connection between Broca and Wernicke areas broken reading aby, ten «. Words comprehended correct but cannot be passed on for speech or writing accompanied by aphasia, 4. Trouble repeating statements Disinguish rom ds (development reading 4 Global weber) 2, Wie lesions in the presylvian speech area by, Both Broca and Wernick areas damaged * Agena acquired incilty « Labored telegraphic speech with poor comprehension towte 4. Trouble repeating statements 5. Teanscosicl 2, Lesion in the prefrontal cortex bs. Capacity to repeat statements unimpaired Patient cannot speak spontancously 174 fiiSdical Organic Disorders G.Brain and Behavior Original Drawing Patient's Drawing Name Localization #33 Perseveration Frontal lobe Constructions Nondominant (ght) spraxia paretal lobe © O = Figure 13-1. Dystunctions on Common Neurologic Exams, 1. Esstal-cartes: global orientation 1. Key functions i. Speech i, Critical to personality Si, Abstract shought ji. Memory and higher-order mental functions Capacity wo initiate and stop tasks vi. Concentration Lesions of dorsal prefontal cortex i. Apathy fi, Decreased driv, initiative ii, Poor grooming i, Decreased attention 1% Poor ability to think abstr i. Broce aphasia (Fin dominant hemisphere) & Lesions of rbitomedil frontal cortex 4. Withdrawal ii Fearfulaess i, Explosive mood jx. Loss of inhibitions Violent outbursts LUSMLE Step 1: Behavioral Sciences 2. Temporal cortex 2. Functions i. Language i. Memory ii, Emotion ', Lesions stom fom stroke, rumor, and trum herpes virus CNS infections often affect temporal cortex «6, Blateral lesions dementia . Lesions of the rostal (front) lft temporal lobe: deficits in reall proper names «. Lesions of dominant lobe i. Buphoria fi Auditory hallucinations i, Delusions iv. Thought disorders 1% Poor verbal comprehension (Wernicke) learning of £. Lesions of nondominant lobe: 4. Dysporia ii. Iritabilty ii, Decreased visual and musical ability 3. Baie cores 4 Rey function: intelectual processing of sensory information i, Left: verbal processing (dominant) |i. Right visual-sptial processing (nondominant) Lesions of dominant lobe: Germann syndrome i. Agraphia fi, Acaleulia fi, Finger agnosia in Right left disorientation +» Dysfunctions in this area sccount for aproportion of leering dissbi- «: Lesions of nondominant lobe: 1. Denil of illness anosognosia) fi. Constuction speai dificulty outlining objects) Ii, Neglect ofthe opposite side (eg, not washing oF dressing opposite de of body) 4. Oscipitalcoe a Key functions 3. Visual input 5, Recall of objects sane and distances PET scans show activity inthis aren ding recall of vial images », Destruction: cortical indness 176 medical «Bilateral occlusion of postesior cerebral arteries: Anton syndrome 4. Conta blindness fi, Denial of blindness <4. Cannot se camouflaged objects «Occipital epleptic foe 5. Limbicsystem 4 Consists of hippocampus, hypothalamus, anterior thalamus, cingulate gyrus, amygdala Associated cortical areas can suppress external displays of internal tates Key functions i, Motivation i. Memory 4H, Emotions (mediation between cortex ane lower centers) ix, Reflex ac for conditioned responses Violent behaviors visual hallucinations i. Socisexual behaviors 4. Associated dysfunctions i. Apathy ii Aggression i, Vegeatve-endocrine disturbances ix. Memory problems and learning new material © Hypothalamus i. Implicated in involuntary internal responses that accompany emo sional steategy ii Regulation of some physiologic responses + Increased heart and respiration + Flevation of blood presure and diversion of blood to skeletal mus cles when angry + Regulation of endocrine balance + Control of eating (hungeritirst centers) + Regulation of body temperature + Regulation of sleep-wake cycle li. Dysfunctions + Destruction of ventromedial hypothalamus: hyperphagia and obesity + Destruction of lateral hypothalamus: anorexia and starvation, & Thalamus i, Critical to pain perception fi, Dysfunctions lead to impaired memory and arousal « Retcular actvating tem (RASL i, Motivation fi Arousal fi, Wakefulnss Organic Disorders LUSMLE Step 1: Behavioral Sciences, hy, Hippocampus: eitical for memory and new lering ‘Table 19-3 Lesions and Memory Tain ‘(Shores onary [TongrmNomory —[NewLewing | eal pol abe |Spacd Spars inpaied | [Hippocampus Spe Tinpait — | i Ameiala i, Dorsomedial portion of temporal lobe fi, Connection with corpus striatum ii, Diret link between limbic system and motor system. ins Critical roe in emotional memory and rudimentary learning ¥. The wi Kltertiney syndrome + Removal of the amygdala + Tame mneonscious mind”= + No fear of natural enemies + Hyperactive sexually «High age threshold Make lve, ot wae” vi, Konakoftayndsoms + Arunsia suing rom conic thiamin decency + Assxiated with alcoholism + Neuronal damage in the tharos + Once neuronal damage the thalames, not eaable with thiamin 6. Basal ganglia ‘Functions: ntation and control of movement, implicated in depression and ernetia b, Dysfanetions (ce Dementia section) 1. Parkinson disease ii Hanington chores i Wion disease iv, Fabr disease + Rare hereditary disorder + Calecation ofthe basal ganglia + Onset tage 30 + Dementia at age 50 + Reembls negative symptom schizophrenia 7. Bons ‘Start of NE pathway B, Important for REM sleep Anomolis here inked to autism 178 inedical L 8, Cerebellum 2. Key for balance », Skillbased memory Facilitates verbal reall, 4. plicated in some learning disabilities | epicOiodes Tarte pone (ripe nat) Heed dere (rae cocepalis rd mel), Achime’ ype dementia (ros conmon ge asscited deme vascular eesti (mutpleceeba inf), ek ease (ey a), Cet dese (orion induced). Huntington chore (eosoal dona mileage ene), nd Parkinson dese (abe comenen in eet, depen deen). | Dementia ae orc and ite evel and mus be dings odes, whi haan sae one and wy reve,“ compare to. has as Weis conduton, gobs nd van aphs. The fice and eet of less inthe ont cate enpr cte pietl cae op cone, nb ster (pncamps hypetams, ane hams, cote aS | i nal agi prs nd cecum ae diced. ade comptes eect on | erorfsosin te mel engrlobe tse in te herons Organic Disorders iédical 179 Psychopharmacology ‘A. Neurotransmitters 1. Acetyicholine (ACN) 1 Neurotransmitter at nerve-muscle connections fr al voluntary muscles ofthe body '. Also many of the involuntary (autonomic) nervous sytem synapses «. Despite long history, the exact role of AC in the Brain unclear 4. Chotinergl neurons concentrated inthe RAS and basal forebrain Significant role in Alzheimer disease f. Dementia in general associated with decreased ACh concentrations in amyg- dala hippocampus, and temporal neocortex fg. Associated with erections in males bh, Masarinic and nicotinic receptors Inthe corpus striatum, ACh circuits aren equilibrium with dopamine neurons 2, Norepinephrine (NE ofthe catecholamine ncurotransmitters smittrof the sympathetic nerves ofthe autonomic nervous system, Which mediate emergency response i, Acceleration of the heart fi. ilstation ofthe bronchi li, Hlevation of blood pressure «. Inpicated in atering attention, perception, and mood 4. Key pathway: lous cevuleus in upper pons Implicted in monoamine hypothesis of affective disorder: i, Depletion of NE lead to depression ii, Exes of NF (and serotonin) leads to mania fi, Based on two observations: + Reserpine depletes NF and causes depression, + Antidepressant drugs block NE re-aptake, thus increasing the amount ‘of NE available postsynapticaly Receptors: i. Alpha-t: sympathetic (easoconstriction) fi. Alpha-2: on cell bodies of presynaptic neurons, inhibit NE release Sik, Beta: excitatory for heat, lungs, brain is, Beta-2 ectatory for vatodilaation and bronchodilatation Note ‘The USMLE Requires You to row. When te dg used + hat ae side es + Managemen of side es + Pay speci tention tothe chugs sted inthe "Drugs to Hight” section USMLE Step 1: Behavioral Sciences 9. Dopamine ‘The other catecholamine neurotransmitter Synthesized from the amino acid tyrosine D, receptors mest important D, and D, stimulate G-protein and increase cAMP and excitation Dz, Dy and D, inhibit G-protein and decrease cAMP and exitation Thre pathways of known peyeiatri importance: 4. Nigrotratal pathway in substantia nigra (movement disorders) + Dopamine blockade va neuroleptics or destruction of dopaminergic cells in Parkinson disease + Lend to Parkinson symptoms (temors, muscle rigidity, bradykinesia) fi, Meso-limbic-cortico pathwey (psychosis) + Portynaptie dopamine blockade produces antipsychotic symptoms ‘Neuroleptic medications have an antidopaminergic effect and reduce the positive symptoms of schizophrenia (hallucinations, delusions) + Dopamine levels are also lower in depression end slightly higher in + Amphetamines cause the release of dopamine and NE and block ‘heir re-uptake, thus aggravating psychotic symptoms. + Tourette's syndrome i also ¢ disorder of a hyperactive dopamine | system, for which haloperidol s sometimes effective | ii, Tuberoinfundibular sytem i + Dopamine sa prolatin-inhibiting factor. + Brolactn levels can serve as rough indicetors of overall dopamine activity i Nucleus accumbens (NAC) pathway + Positive reinforcement occurs here. + The addiction pathway” 4, Serotonin (5-hydroxytryptamine, HT) 4. The tanamiter of «discrete group of neurons that all have call bodies located i inthe raphe nuclei ofthe brain tem 1, Changes in the activity of serotonin neurons are related to the actions of| papchedelie drugs. «Involved inthe therapeutic mechanism of action of antidepressant treatments (amost are 5-HT re-uptake inhibitors: afew new ones are 5-HT agonists) 4. Has inhibitory influence linked to implse control Law $-HT = low impulse control f. Hasrol in regulation of mood, sep, sexual activity, aggression, anxiety motor activity, cognitive function, appetite, ctcadian rhythms, neuroendocrine func tion, and body temperature «Involved in various neuropsychiatric disorders, eg, depression, mania, impul- sive aggression, schizophrenia, anxiety, obsessive- compulsive disorder, autism, Aleheimer disease, chronic pan, anoresa, bulimia, narcolepsy, myocionus 1, Characteristics in blood platelets ae good model forthe brain neepee 182 fiiedical 5, Glutamic acid 1 1. One of the major amino acids in general metabolism and protein symthess also Stimulates neurons to ire principal excitatory neurotransmitter inthe brain «. The neurotransmitter ofthe major neuronal pathway that connects the cerebral cortex and the corps striatum, 4, Also the transmitter of the granule cells which are the most numerous neurons inthe cerebellum «Evidence tat glutamic acids the principal neurotransmitter ofthe visual path= May have a zoe in producing schizophrenic symptoms Reason for PCP symptoms (antagonist of NMDA glutamate receptors) hi Glutamate agonists produce seizures in animal studies 6, Enkephalins a. Composed of two peptides each containing five amino acids ', Normally occuring substances that act on opiate receptors, mimicking the effets of opiates «. Neurons are localized to areas of the rain that egulate functions influenced by opiate drugs. 2. Substance P 2 Peptide containing 11 amino acids . A major transmitter of sensory neurons that convey pain sensation from the periphery especially the skin, into the spinal cond «Also found in numerous brain regions ‘Opiates relieve pain in part by blocking the release of substance P. «New cass ofentdepressant medications being tested to work on substance P 8. Gamma amino-butyric aid (GABA) ‘4. One ofthe amino-acid transmitters inthe brain 2, Occurs almost exctsively in the brain « Reducrs the fring of neurons; principle inhibitory neurotransmitter in the brain 44. The transmitter present at 25 o 40% ofall synapses inthe brain Quantitative, the predominant tansmitr inthe brain £ Associated with anxiety, cannabis, benzodiazepines B. Antipsychotics (Neurolepics) 1, Treatment concerns ‘4. Most common cause of relapse is nonadherence ' Most common reason for lure of treatments inadequate dosage «. Worse behavioral symptoms on antipsychotics, check for undiagnosed organic condition 2, Common uses ‘a. Schizophreni: thought disorders, hallucination, alterations of affect, autism, ideas of reference medical 185 USMLE Step 1: Behavioral Sciences Note erapyraial eactons + Choro: jeky movements + Rhetod sow, coniewous movements + Rhythmic stereotypical movements 184 medical ’, Any general psychosis, regardless of diagnosis i, Alcoholic hallucinoss fi. Amphetamine psychosis Organ psychiatric disorders agitated dementia: can cor deliam 44 Tourette syndsome: haloperidol, pimoride, clonidine ‘e. Movernent disorders: Tourette, Huntington, and hemibalism (Ialing move ments) Nausea and vomiting : Intractabl hicups reat agitation in dementia 1, Protas Involuntary medication of chronically ill psychiatric pasients who persistently reise treatment doesnot result in ealier release, more cooperation, or more insight Mechanisms of action 4 Dopamine blockage at postsynaptic reeptors by Alpha-adrenergic blockade therefore, hypotensive effect .Antichotnergc action by blocking the muscarinic receptors 4. Blocks both NE re-uptake and serotonin and histamine receptors Adverse effects a. Neurologic effects i. Anticholinergic effets: very common, effects additive if given with ‘other anticholinergic agents + Dry mouth + Bluey vision + Constipation + Urinary retention Delirium Memory sid: "blind as bat, dey 25a bone, red asa beet, mada a hatter" Especially frequent in the elderly i, CNS effect from antagonism of i receptors * Weight gin Sedation very common + Impaized memory ‘il, Extrapyramidal (EPL reactions: due to decreased dopamine; appear in ‘one-balf ofl patients in irs few months «Treat with anti-Parkingon drugs, amantadine, benztropine “Table 14-1. Extrapyramidal Reactions to Antipsychotic Medications Side Effects Peak DDystonie reactions (jerky movernents, rouble speaking) | 1 week (younger are more at risk) ‘kines weeks Riglity Bweeks Tremors weeks ‘Abathisia TO weeks and Rabbit syndromes 184 weeks iw. Taive dyskinesia (TDY the primary reason for nonadherence to ‘weatment + Rarely before 3f0 6 months, ! month if older than 60 + Occurs in 20 to 359 of patients * Sen tone rotons tremor and spas of enc band + Perssts after medications are terminated (5 to 10% remit; ineapac- tating in 5% of eases ‘Cause: supersenstivity of postsynaptic dopamine receptors + Predisposing factors include older patients, long treatment, smoking, iabetes mellitus + Symptoms do not occur during sleep + Suppressed by voluntary movements fr short ime (versus cerebellar Aisease tremor, which worsens with intentional movement) + Streisand movements in other body parts aggravates + AnticParkinson drugs aggravat + No treatment, focus on prevention: pimozide or loxapine has les chance of inducing TD, clozapine not associated with TD ata 1b Non-neurologc effects 4. Cazdiorascula ffs orthostatic hypotension (do not use epinephrine, lowers blood pressure further) 4, Particular taste (also dental cavities) iil, Vomiting common with long-term use, especially among smokers iv, Sexual effets: prolactin elevated + Men: decreased libido, inhibition of ejaculation, retrograde ejaculi- tion + Women: breast enlargement and lactation, changes in libido Ales body responce to temperature LUSMLE Step 1: Behavioral Sciences, ‘Table 14-2, Potency of Amipsychotic Medications [eeey TatepyramialSjmpioms[ Anicbolineac Boas | High (haloperidol) High Low ‘ow ehorromrine) Tw igh ete 6 Potency ofthe anchor is ime elated tos anchor ef. ~—— 7. In general, low-potency antipsychotics have fewer EP side effects, a greater sedative ‘Drugs to Highlight ‘effect, a greater hypotensive effect, and more anticholinergic side effects, |. Helopesia isthe most used for Teta grea Sow Laue “able 143 Typical vers Arp Antieychatie en ese (nicl ‘pial + Revogate geossion ‘Dopamine Dopamine and serotonin finer Tres monly posvesympions | Treats pstine and tne moms Soupresaaaee | Movesie tt Fevers es + Weak rention cer 02 recaps ~ ™ Tiare |G aprons = + Orta 306 secs 1, Common uses + Als ened pte see + Serta amie (eres) a ‘anes et ast ™ + sass iT. peor ors es, 4 ioe + Chute tw slo Hasse ad epic Srpums eu ses + See es ie yah ec, Inpse wor + tases poacnlac a 5 Cirapie + sors veo eptie Srp eh rs + Usui tet tee rp ets 186 medical 4 Depresion (any drug) 1. Enuresis (imipramine) «Panic disorder (imipramine) 4, Obsessive-compulsive disorder (Suoxsine clomipramine) ‘Chronic pai, with and without depression (amitriptyline) 2, Cyclic antidepressants 1. For severe, hospitalized patients Action: blocking of re-uptake of serotonin and norepinephrine, blocking of| alpha-l adrenergic receptors, and muscarinic receptors «Pharmacokinetics i. Fatsoluble i Merabolized by the liver and excreted by the kidneys ii, Requises ceeching plasma levels for imipramine, nortriptyline, {esipramine, and amitriptyline for efficacy 4. Adverse eects i. Antichoinergc eects (se Antipsychotics section) i CNS ficas + Drowsiness + Insomnia and agitation + Disorientation and confusion + Headache + Fine tremor fi, Cardiovascular from antagonism of alpharl adrenoreceptors and Inhibition of -HT re-uptake + Most common in elderly + Tachyeardia + Orthostatic hypotension: managed by sodium choride tablets, caffeine, support hose or biofeedback jw. Seoul + Men: impotence testicular swelling + Women: anorgasmia and breast enlargement (teat with cyproheptadine) ‘y Metabolic: changes in blood sugar levels © Cautions i. fective in only 70% of depressed patients li. Motor activity increased before depression affected: window of Increased suicide rise li, Not for patients with respiratory difficulties: ies up bronchial seere- tions iu. May lower seizure threshold 1 May impaie driving vi. Potentiates effet of alcohol vil, Manic episode induced in 50% of bipolars vill. Avoid daring fist trimester Jc Baby gets 19% of mothers dose in breastmilk Withdrawal 4. After prolonged us, should be gradual Ti Akathsi, dyskinesia, anxiey, sweating, dizziness, vomiting, choline sc rebound, depression rebound 3. Selective serotonin re-uptake inhibitors (SSRI) Most widely used ansidepresants Also used for obsessve-compulsve disorders, Tourette syndrome « Nofect on NE or dopamine, very selective blockage of re-uptake of serotonin 4. Fewest adverse effects of any antidepressants currently available so the legest seling ce Adverse effects 4. Anorgasmia and delayed orgasm in 15 to 20% of patients fi, Serotonin symdrome + Associated with: high doses, MAOI and SSRI combo, MAOT and syathetie narcotic combo + Symptoms general restlessness, sweating, insomnia, nausea, diar- shea cramps, deirom + Treatment: remove causative agen, reduce SSRIs, give eyprohepta~ dine £ Drugs from this class 4, Fluoxetine: most commonly prescribed 4 Sertraline: metabolized faster by liver and remains in blood shorter tiene fi, Paroxetine: most potent ilucal 187 LUSMILE Step 1: Behavioral Sciences Note ‘rugs to Highlight 1 ioe + bites tse + Ais etree meres + Seng bitte intake au os reteset ep + Moyet ea thee somee do Nea aeeasnon eye Sw Ea SH ee oc sang $2 veo, + Ses tee ese (rn ne oie, veaiign 3 open + Wesabe cere ode ele on Mn at serene Noswidatreycett Lil crdac dares Newsee owen Sie ects pte sirpese ob, 4 tne + this eagle oth ards Ht raseponre ted + Acetic 556 1+ Sextet: sig ns, ‘ena se, ‘emt, secon rs, pee 5 ene + sips mets eapna nema Peat rape + tie ea onabra zr vague + Orbe gs + Siete: dane ou crt beer 188 médical i, Flevoxamine: causes sedation 1 Gitalopram: lowes cost per dase 4. Monoamine oxidase inhibitors (MAOIs) a. Mechanism of action i. Inhibits MAO, an enzyme that metabolizes serotonin, epinephrine, and NE i, For best effet, reduce MAO activity by 809% by, Adverse effects i. MAOI + TYRAMINE - HYPERTENSIVE CRISIS + Problem foods: cheese, dried fish, sauerkraut, sausage, chocolate, avocados + Safe fonds cottage cheese, some wines + Signs: occipital headache, stiff neck, nausea and vomiting, chest pain, dated pupils, nosebleed, elevated blood pressure + Treatment stop medication, gve phentolamine alpharblockage) oF chlorpromazine (antipsychotic with bypotensive elects) D. Flectroconvulsive Therapy (ECT) 1. Common uses a. Depresion (80%) Db. Schzoaffectve disorder (10%) Bipolar disorder 2, Mechanism of action a. Electricity is passed fom the frontal cortex to the stratum. ', 80H show some immediate improvement Unully requires 5 to 10 treatments 4. Teats episodes, not fr prophylaxis «€. Only contraindication is increased cranial pressure (eg tumor) 5. Side efecs ‘4. Ancithesia cliinates fractures and anticipatory anxiety. 'b, Memory loss and headache common, returns to normal in several weeks «5. Serious complications <1:,000 4, Other sss 4. Altnough not usually Gstine treatment, shouldbe considered for: i. Highly suicidal patients ii, Depressed pregnant patients '. Improvement associated with lage increase in slow wave (dela) activity in the prefrontal aes; greater increase = greater recovery a. For long-term control and prophylaxis of bipolar disorder, migraine cluster headaches, chronic aggression; combined with tricyclics for resistant depression 1, Works for 70% of cases «. Hypothesized mechanism: blocks inositol-1-phosphate (second messenge) 4, Pharmacokinetics: i. Quickly absorbed from the gastrointestinal tract, not protein bound or metabolized 4, Requites reaching plasma levels very dose to toxic levels for effet, Wich is eached in 10to 14 days ii, Mast monitor blood levels + Therapeutic levels:08-1.5 mEq, + LA m/l may be tox + rank toxicity st 20; above 2.5 = hemodialysis Ji. Good kidney function and adequate salt and uid intake esental 95% excreted in urine 1 Peak serum love: 1-3 hours vi. Potassium-sparing diuretics have no effets; loop diuretic result in increased serum levels Side effects i. Narrow margin of sifety. must monitor blood levels fi, Themon hist anorexia, gstmintstnal distress commonly occur at therapeutic levels vie Beniga eukcytois vi, Hpotheraidisa {Long-term lithium use as adverse efcts on renal function. i 4 Compliance ofen dificult, patient may value manic experiences 1 Teratogeic produces cardiac malformations (Ehstein anomaly tricuspid valve) 2. Valo acid ‘2. For acute mania, rapid cyelng bipolar disorder, impulse control 'b. Mechanism of action: augmentation of GABA in CNS «6 Monitor biood levels 4. Hepatotosic liver function impaired) Side fics i. Sedation fi, Mild tremor i. Gastrointestinal distress fe. Occasional agranuboestosis 1. Attonic levels confusion, coma, cardiac arrest Btiical 169 USMLE Step 1: Behavioral Sciences ‘3. Carbamazepine: Secondclin teatment a For acute mania, rapid eyeing bipolar disorder, impulse contol b. Mechanism of action os with action potential ii Alters central GABA reeptors Monitors blood levels and signs of rash 4. Side effec i, Similar to valproic acid plus nausea, rash, mild leukopenia li, Occasional agranulocytosis Tone levels: atrioventricular block, respiratory depression, coma . Anxiolytics(Antianaity) 1, Benzodiazepines Used for ancy, acute and chronic aleohol withdrawal, convulsions, insomnia, “restless leg akathisia, panic disorder cptesce CNS at limbic system, RAS, and corte , Mechanism of action ote Binds to GABA chloride ceptors aes acon of GABA , 4. Pharmacies Oras Hit Tal undergo epatemicosol eatin, ee fo laepan 1 sper resp snd cry, hc ne cone cnugeon + Fray i Weltaoted oral + Roarcomartr . Aiwee fc rmusclereaxng i. CNS depression (sedative eect) ports + Aft serotonin, not ii Paradoxical agitation Ii, Confusion and disorientation, especialy i elderly ctBs ji, Overdose: apnea and respiratory depression (not for ws with patients + Tay tor eect ‘with sleep apace) + Some sedation \ Withdeawat: insomnia, agitation, anxiety rebound, gastrointestinal + Low abuse pote liste abrupe withdrawal can bring om seizures + No witha fs . Diminishes effectiveness of ECT. + Not potent by : Lowers tolerance to alcohol alcohol 1 Crosies placenta and accumlates in fetus, withdrawal symptoms in newborn 2 Venlefxine 3. Passed on i breastmilk with observable effects «+ Anidepresat eerty 5. Oral contraceptives decrease metabolism of benzodiazepines approved for enealved amity dsrder 1 ibis reuptake of NE and 547, mild dopamine eet + 4 weeks for feet 190 inédical ‘Tuble 14-4, Selected Benzodiazepines and Common Uses [Generic Name ‘Common Uses 1 (Faster acting | | Dine Anse imomsia | | rsespam tovoia | | “Triazolam Insomnia 1 Longer acting Alprazolam Pani oot Chlodinrepoxide cok detoaation Clonsepan Pai aniety “emavepam Insonsia Loctzepam Anxiety ool zed eres ‘Major neuovansiters: acechole, norepinephrine dopamine setctonin, hzaic ac, | keris substance Pang sprna amino uy a pyc ups ues sie efes and management side cls for common sed pa od | ypc agents are reviewed, Pay specl ateton to "Drags o Highlight” Antidepressant drugs uses sid ec, and management of delet for commanly used ec, ‘SSRs and MAO are reviewed, Pay speci atetion to "Drugs to Highlight” Becroconwisat Therapy (EC) used mainly io teat depression, conser strongly for depressed sci pains Mood Sabie: wes and cautions for lito, voi ac, end carbamazepine ae reviewed Asics (atin) uses, mechanisms of acon, and side feds ofthe benzodarepines and | ene agents are reed. Pay speci tention to “Drugs to Highlight” Psychopharmacology 191 LUSMLE Step 1: Behavioral Sciences 192 fiiédical Review Questions 108, Following his atest for disturbing the peace, a 35-year-old man ie refered for psychiatric valuation. During the initial interview, he talked rapidly and paced around the room. ‘He reported that head slept ite in the past few days, but tat he felt “great. “Its not aways like this” he confided. "Sometimes I jus feel so bad I can hardly move” During ‘the most common subsequent treatment for this disorder, which of the following side effects wold the patient be aot likely to experience? (A) insomnia (@) memory loss (©) tardive dyskinesi (D) acne (@) retarded ejaculation 109, A 36-year-old woman is brought to see her physician by her family. The family reports ‘that over the pas year she has had trouble controlling her movernens. Symptoms include frequent and discrete brisk movements that caus jerks ofthe peli an limbs a8 well as facial frowns, grimaces, and smirks, The most likely diagnosis fortis patient would be (A) Pick disease (B) Wilson disease (C) Parkinson disease (D) Huntington disease (B) Creuafeldt Jakob disease 110, Over 5.week period, a previously healthy 55-year-old female has developed headaches, progresively severe word-finding dificult, and confusion. She speaks incoherently and |s unable to fllow commands, repeat phrases, of name objects. What is the most likely site ofthe lesion? ) frontal lobe (B) temporal lobe (©) occipital lobe (D) parietal lobe (@) cerebellum 1. Following an automobile accident, 46-year-old males brought ta the hospital suffering from head trauma. Over the next week, is medical record nots the appearance of au ‘ory hallucinations delusions, thought disorders, and poor verbal comprehension, These symptoms are most consistent with a lesion inthe (A) front fobe (8) dominant parietal lobe (©) nondominant parietal lobe (0) dominant temporal lobe (&)nondominant temporal lobe 112. A.32-yearold male presents at the local clinic complaining of abdominal cramps, sweat ‘ng, runny nose, vomiting, and muscle aches. Eeamination shows that his pulse is rapid and pupils are diated. He sats that he“fels ust awful” and that he has hd these 5 mp tom for about 24 hours. The mos lkaly pharmacologic treatment would! be (A) none (@) diazepam (©) carbamazepine (D) clonidine (@) twarodone 113. A 58-year-old chronic alcoholic is evaluated and found to have difficulty with recall of fecent evens although long-term memories seem intact. The patient appears confused and init that he has met the physician before, akthough thisis not the ease. These symp toms are the result of syndrome that can result from neuronal damage othe (a) cereballum (8) hippocampus (©) bypothalarnas (D) reticular activating system () thalamas 114, A 32-year-old schizophrenic patient has been taking a standard course of neuroleptic medication forthe past year. This medication has been very effective at controlling his delusions and hallucinations. However, during a regular check-up the patient was found to be sffring from dry mouth and constipation. The patent report that it shard to read because his vision is blurry, and then lapses into delirium. These spmptoms are most likely produced by blockage of what receptor? (A) dopamine (8) histamine (©) muscarinic (D) norepinephrine ) serotonin 115, A 24-year-old woman was recently diagnosed with undiferentiated schirophrenia and placed on a standard drug tegimen, About 2 weeks ito her treatment she experiences fisturbing extrapyramidal side effects, The most likely side effect to appear at this time would be (A) abathisia (8) akinesia (©) physical wemors (©) Pisa syndrome ) tandive dyskinesia medical 195 USMLE Step 1: Behavioral Sciences U6 Pits ped on wish of th allowing medion mot ily sos bare Blood mai checked on a weely basi? (A) dlosapine (B) aloperidel (©) olanzapine (b) risperidone (B) thioridazine 117. A.46-year-old man who was being teated for depression is brought to the emergency {department complaining of headache atthe base of the sll, chest pains and stiff neck, Physical exam shows that his nose is bleding, his pupils are diated, and his blood pres sues extremely high. He tepors voriting repeatedly in the past few hours. Based on {his initial presentation the physician suspects thatthe patient has recently esten which ofthe following foods? (A) avocados (8) cottage cheese (©) hamburger (D) raw one (french res (2) fied chicken (G) oie figs r Answers 108. Answer: D.Lithiunn has been associated with side effect leading to lethargy (hypothy- roids), edema (lithium isa salt) acne, and seizures. 109. Answer: D, Age of onset and description indicate the onset of Huntington chorea. 10. Answer:B, Temporal lobe is associated with nguage, memory. and emotional expression 111, Answer: D, Lesions ofthe dominant temporal lobe result in euphorie, auditory halluc ‘ations, thought disorder, and poor verbal comprehension. 112, Answer: D. The man is suffering from opiate withdrawal. The drug on thelist to treat the withdrawals clonidine 113, Answer: E. The man mos likely is suffering from Korsako syndrome in which chronic thiamin deficiency leeds to neuronal damsage in the thalamus as well asthe frontal lobes. LL, Answers. The patient displays the symptoms of anticholinergic intoxication. This isthe result of blockage of the muscarinic reeptors. 11S. Answer: B. Difficulty completing simple movements or common skils occurs relatively carly in the course of treatment, Tremors ad akathisa appear after about to 2 months, “Tadive dyskinesia rarely occurs before the patient is on the medication for atleast 3 ronths 116, Answer: A. Because ofthe risk of agranulocytosis with clozapine, blood must be moni tored weekly 117, Answer: A. The patents suffering fom a hypertensive criss tat is key the result of ‘consuming tyrmine-contsining food while on MAOIs for his depression. Tyramine ca be acquired fom cheese, deed fish, snerkraut, sausage, chocolate, or avacadas ___Review Questions Ethical and Legal Issues | | | | Karen Ann Quinlan: Substituted Judgment Standard Inthe Quinn case Karen Ann was na pestent vega ae beng kept le ont bye support Karen's fate sted io hae ere supp terminated acocing to is undestaning of ‘what Kaen an weuld want. The court eel at Kate here were mirauousy i for a9 intenal.and perceptive of her ievrsbe condo, she cou efectely decide upon

You might also like